Об интеллектуальной собственности Обучение в области ИС Обеспечение уважения интеллектуальной собственности Информационно-просветительская работа в области ИС ИС для ИС и ИС в области Информация о патентах и технологиях Информация о товарных знаках Информация о промышленных образцах Информация о географических указаниях Информация о новых сортах растений (UPOV) Законы, договоры и судебные решения в области ИС Ресурсы в области ИС Отчеты в области ИС Патентная охрана Охрана товарных знаков Охрана промышленных образцов Охрана географических указаний Охрана новых сортов растений (UPOV) Разрешение споров в области ИС Деловые решения для ведомств ИС Оплата услуг в области ИС Органы по ведению переговоров и директивные органы Сотрудничество в целях развития Поддержка инновационной деятельности Государственно-частные партнерства Инструменты и сервисы на базе ИИ Организация Работа с ВОИС Подотчетность Патенты Товарные знаки Промышленные образцы Географические указания Авторское право Коммерческая тайна Академия ВОИС Практикумы и семинары Защита прав ИС WIPO ALERT Информационно-просветительская работа Международный день ИС Журнал ВОИС Тематические исследования и истории успеха Новости ИС Премии ВОИС Бизнеса Университетов Коренных народов Судебных органов Генетические ресурсы, традиционные знания и традиционные выражения культуры Экономика Гендерное равенство Глобальное здравоохранение Изменение климата Политика в области конкуренции Цели в области устойчивого развития Передовых технологий Мобильных приложений Спорта Туризма PATENTSCOPE Патентная аналитика Международная патентная классификация ARDI – исследования в интересах инноваций ASPI – специализированная патентная информация Глобальная база данных по брендам Madrid Monitor База данных Article 6ter Express Ниццкая классификация Венская классификация Глобальная база данных по образцам Бюллетень международных образцов База данных Hague Express Локарнская классификация База данных Lisbon Express Глобальная база данных по ГУ База данных о сортах растений PLUTO База данных GENIE Договоры, административные функции которых выполняет ВОИС WIPO Lex – законы, договоры и судебные решения в области ИС Стандарты ВОИС Статистика в области ИС WIPO Pearl (терминология) Публикации ВОИС Страновые справки по ИС Центр знаний ВОИС Серия публикаций ВОИС «Тенденции в области технологий» Глобальный инновационный индекс Доклад о положении в области интеллектуальной собственности в мире PCT – международная патентная система Портал ePCT Будапештская система – международная система депонирования микроорганизмов Мадридская система – международная система товарных знаков Портал eMadrid Cтатья 6ter (гербы, флаги, эмблемы) Гаагская система – система международной регистрации образцов Портал eHague Лиссабонская система – международная система географических указаний Портал eLisbon UPOV PRISMA UPOV e-PVP Administration UPOV e-PVP DUS Exchange Посредничество Арбитраж Вынесение экспертных заключений Споры по доменным именам Система централизованного доступа к результатам поиска и экспертизы (CASE) Служба цифрового доступа (DAS) WIPO Pay Текущий счет в ВОИС Ассамблеи ВОИС Постоянные комитеты График заседаний WIPO Webcast Официальные документы ВОИС Повестка дня в области развития Техническая помощь Учебные заведения в области ИС Поддержка в связи с COVID-19 Национальные стратегии в области ИС Помощь в вопросах политики и законодательной деятельности Центр сотрудничества Центры поддержки технологий и инноваций (ЦПТИ) Передача технологий Программа содействия изобретателям (IAP) WIPO GREEN PAT-INFORMED ВОИС Консорциум доступных книг Консорциум «ВОИС для авторов» WIPO Translate для перевода Система для распознавания речи Помощник по классификации Государства-члены Наблюдатели Генеральный директор Деятельность в разбивке по подразделениям Внешние бюро Вакансии Закупки Результаты и бюджет Финансовая отчетность Надзор
Arabic English Spanish French Russian Chinese
Законы Договоры Решения Просмотреть по юрисдикции

Индия

IN053

Назад

Manual of Patent Office Practice and Procedure, 2010, © Office of the Controller General of Patents, Designs & Trade Marks


DRAFT

Manual of Patent Office Practice and Procedure

PUBLISHED BY THE OFFICE OF CONTROLLER GENERAL OF PATENTS, DESIGNS & TRADEMARKS

BOUDHIK SAMPADA BHAWAN, S M ROAD, ANTOP HILL, MUMBAI (INDIA)

Preface

The Patents Act, 1970 was amended in 1999, 2002 & 2005. The amended Act, in accordance with TRIPS, has provided for product patents in foods, medicines and chemical substances. India became signatory to PCT in 1998. As a consequence, patent filing including PCT National Phase Applications have increased exponentially.

Considerable changes have been made in the patenting procedure through the introduction of Patent Rules, 2003, which were further amended in 2005 and 2006, resulting in new practices and procedure.

Indian Patent Office has been modernized with office automation and electronic processing of patent applications during the past one decade. The objective of the modernization project was to increase the efficiency and streamline the functioning in tune with the international scenario, especially with regard to transparency. It has been our constant endeavor to be more transparent in all the spheres of patenting process. IT enabled office process have enabled us to make all information relating to the status of patent application and contents of file, publicly available through the official website. This manual is yet another step to fulfill our commitment to be more efficient and transparent.

There has been a demand from the new inventors, patent practitioners and foreign applicants to codify the practices in Indian Patent Office in the form of a manual. A draft manual was published in 2009 in the official website inviting comments and suggestions from stakeholders. Taking into account the comments and suggestions received, that draft has been thoroughly revised to make it more precise and simple. It is our ardent hope that this manual will effectively meet the demands from stakeholders.

This manual may be considered as a practical guide for effective prosecution of patent applications in India. However, it does not constitute rule making and hence does not have the force and effect of law.

The manual will be revised from time to time based on interpretations by a Court of Law, statutory amendments and valuable inputs from the stakeholders.

(P H Kurian) Controller General of Patents, Designs and Trademarks

INDEX

Chapters:

01 Introduction 02 Key definitions

02.01 General

02.02 Definitions

02.02.01 Controller

02.02.02 Government undertaking

02.02.03 Invention

02.02.04 Inventive step

02.02.05 Capable of industrial application

02.02.06 Legal representative

02.02.07 Assignee

02.02.08 Patentee

02.02.09 Patent office

02.02.10 Appropriate office

02.02.11 Person

02.02.12 Person interested

02.02.13 Prescribed

02.02.14 Prescribed manner

02.02.15 True and first inventor 03 Filing of Patent Applications

03.01 Applicant

03.01.01 Procedure to be followed in case of death of applicant or in case the legal entity ceases to exist

03.02 Jurisdiction

03.03 Type of patent applications

03.04 Filing of a patent application

03.04.01 Contents of Patent Application

03.04.02 E-filing

03.04.02.01 Steps for e-filing of Patent Application

03.04.03 Leaving and serving documents at Patent Office
03.04.04 Receiving documents in Office
03.04.05 Language and Paper size etc.
03.04.06 Sequence listing
03.04.07 Fee

03.05 Processing of Application

03.05.01 Initial processing

03.05.02 Scrutiny of application

03.05.03 Secrecy Directions and consequences thereof

03.05.04 Inventions relating to Atomic Energy

03.06 Withdrawal of patent application 04 Publication

04.01 Publication of Patent Application

04.02 Early Publication

04.03 Particulars of Publication

04.04 Effects of Publication 05 Provisional and complete specification

05.01 Specification

05.02 Provisional Specification

05.02.01 Provisional Specification – General

05.02.02 Contents of Provisional Specification

05.03 Complete Specification

05.03.01 Complete Specification shall

05.03.02 National phase applications

05.03.03 Title

05.03.04 Field of the Invention and use of Invention

05.03.05 Prior Art and problem to be solved

05.03.06 Objects of the Invention

05.03.07 Summary of the Invention

05.03.08 Detailed Description of Invention

05.03.09 Drawings

05.03.10 Abstract
05.03.11 Best Method
05.03.12 Claims
05.03.13 Unity of invention and clarity of claims
05.03.14 Significance of Claims
05.03.15 Scope of claims
05.03.16 Structure of claims

05.04 Priority of a claim

05.04.01 General

05.04.02 Divisional application

05.04.03 Convention application

05.04.04 Effect of Priority Date of a Claim 06 Divisional application and patent of addition

06.01 Divisional Application

06.01.01 General

06.01.02 Contents

06.01.03 Priority

06.01.04 Appropriate office

06.01.05 Reference

06.01.06 Fee

06.02 Patent Of Addition

06.02.01 General

06.02.02 Novelty and inventive step

06.02.03 Reference 07 Convention application, international application and national phase application

07.01 Convention application

07.01.01 Paris Convention and WTO Agreement

07.01.02 Convention Country

07.01.03 Convention Application

07.01.04 Documents to be submitted

07.01.05 Multiple priorities

07.01.06 Other conditions

07.02 International Application under PCT

07.02.01 PCT International application by Indian applicant
07.02.02 Indian Patent Office as receiving office
07.02.03 Requirements of PCT International Application
07.02.04 Functions of Indian Patent Office as Receiving Office
07.02.05 PCT Fee
07.02.06 International Search
07.02.07 Withdrawals in International Application
07.02.08 International Preliminary Examination

07.03 PCT National Phase Application

07.03.01 General

07.03.02 Basic Requirements to enter National Phase in India 08 Examination and grant

08.01 Request for Examination

08.02 Reference for Examination

08.03 Examination of application

08.03.01 Report of Examiner

08.03.02 Search by the Examiner

08.03.03 Novelty

08.03.04 Inventive step

08.03.04.01 General principle

08.03.04.02 Determination of inventive step

08.03.04.03

08.03.05 Industrial Applicability

08.03.06 Inventions not patentable

08.03.06.01 Section 3(a)

08.03.06.02 Section 3(b)

08.03.06.03 Section 3(c)

08.03.06.04 Section 3(d)

08.03.06.05 Section 3(e)

08.03.06.06 Section 3(f)

08.03.06.07 Section 3(h)

08.03.06.08 Section 3(i)

08.03.06.09 Section 3(j)

08.03.06.10 Section 3(k)

08.03.06.11 Section 3(l)

08.03.06.12 Section 3(m)

08.03.06.13 Section 3(n)

08.03.06.14 Section 3(o)

08.03.06.15 Section 3(p)

08.03.07 Sufficiency of Disclosure

08.03.08 Unity of Invention

08.04 Consideration of the Report by Controller and issuance of FER
08.05 Post-dating of Application
08.06 Pre-Grant Opposition
08.07 Grant of Patent

08.07.01 Compliance of conditions under the Act

08.07.02 Consequences of grant

08.07.03 Date of Patent

08.07.04 Conditions subject to which a Patent is granted

08.07.05 Rights of Patentee

08.07.06 Rights of co-owners

08.07.07 Patent rights not infringed in certain cases

08.07.08 Term of Patent 09 Post-grant opposition

09.01 Post-grant opposition

09.02 Constitution of Opposition Board

09.03 Hearing 10 Post-grant procedures

10.01 Maintenance of Patent

10.02 Restoration of Lapsed Patents

10.02.01 Procedure for disposal of application for restoration

10.03 Registration of assignments/Transfer of Right

  1. Surrender Of Patents
  2. Appellate Board and Appeals
10.05 Working of Patents
10.06 Amendments after the grant of patents
10.07 Procedure to be followed, after the grant of patent, in case of death
of an applicant
10.08 Register of Patents
10.09 Rectification of register of patents
11 Appeals

11.01.01 Appellate Board

11.01.02 Appeals

11.01.03

11.02 Appeal procedure 12 Revocation of patent

12.01 Revocation of Patent

12.01.01 Revocation before High Court or Appellate Board

12.01.02 Revocation by Controller on direction of Central Government

12.01.03 Revocation by Central Government

12.01.04 Revocation by Controller for non-working 13 Compulsory licensing

13.01 Working of patents - General principles

13.02 Compulsory License

13.02.01 Compulsory license under Section 84

13.02.01.01 Reasonable requirements of the public

13.02.01.02 Contents of application

13.02.01.03 Procedure

13.02.01.04 Terms and conditions

13.02.01.05 Application by licencee

13.02.01.06 Compulsory license in case of two or more patents held by the same patentee

13.02.02 License for related patents

13.02.03 Compulsory licence on notification by Central Government

13.02.04 Compulsory licence for export of patented pharmaceutical products in certain exceptional circumstances

13.02.05 Termination of compulsory license 14 Use of patent for purposes of Government

14.01 Use of patent for the purpose of Government

14.02 Power of Central Government to use inventions

14.03 Rights of third parties

14.04 Acquisition of inventions

14.05 Reference of disputes to High Court 15 Patent agents

15.01 Patent Agents

15.02 Disqualifications for registration as a patent agent

15.03 Rights of patent agents

15.04 Subscription and verification of certain documents by a Patent Agent

15.05 Restrictions on Practice as Patent Agents

15.06 Power of Controller to refuse to deal with certain agents

15.07 Power of Controller to remove the name of a Patent Agent

15.08 Restoration of names of Patent Agents 16 Offences and penalties

06.01 Contravention of S.35 or 39

06.02 Falsification of entries in register, etc.

06.03 Unauthorized Claim of Patent Rights

06.04 Wrongful use of words, "Patent Office"

06.05 Refusal or failure to supply information

06.06 Practice by non-registered persons

06.07 Offences by Companies 17 General powers of Controller

17.01 Powers of a Civil Court

17.02 Awarding costs

17.03 Review

17.04 Petition for obviating an irregularity

17.05 Mention of inventor in Patent

17.06 Directions not otherwise prescribed

17.07 Exercise of discretionary power by the Controller

17.08 Power of Controller to correct clerical errors, etc.

17.09 Ex-parte decision 18 General services

18.01 General Services

18.02 Official Journal

18.03 Providing information

18.04 Inspection and supply of copies of documents

18.05 Annual report of the Patent Office

18.06 Information available at the Website 19 Scientific advisors

19.01 Scientific Advisors

19.02 Roll of Scientific Advisors

19.03 Qualifications and procedure for enrollment

19.04 Removal from the roll 20 Miscellaneous provisions

20.01 Affidavits

20.02 Exhibits

20.03 Officers and employees of Patent Office – duties 21 Time limits

20.01 Time limits

20.02 Petition for extension of time

20.03 Time limits prescribed by the Patents Act, 1970 and Patents Rules, 2003 22 Annexure I – First Schedule 23 Annexure II – Second Schedule 24 Annexure III – Fourth Schedule

Chapter 1

Introduction

  1. This manual has been compiled with an intention to codify the practices and procedures being followed by the Indian Patent Office and is intended to serve as a procedural guide for the practitioners and other users of the Indian Patent System. Judicial decisions on the procedural matters have also been included.
  2. Indian Patent Office works from four locations viz. Delhi, Mumbai, Kolkata and Chennai with defined areas of territorial jurisdiction. Introduction of office automation and electronic processing of patent applications has resulted in a certain level of uniformity and transparency. Maximum amount of information has been made available to the public viz. information relating to patent applications, including the status, examination reports and other documents. This manual is expected to bring in further transparency and uniform practices in the four offices.
  3. Processing of a patent application is a multi-stage process, involving filing of an application, electronic data processing, screening and classification, publication, examination, pre-grant opposition, grant / refusal. Different persons like Examiner of Patents, Controller of Patents, Applicant / Agent and general public are involved in the process. Rights, functions and responsibilities of all stakeholders have been explained to ensure smooth functioning of the Patent System.
  4. This manual is expected to spell out patent office practices and procedure and bridge any information gap that currently exists in this regard. It is not intended to be an interpretation of the Indian Patent Law.
    1. The procedure of filing of a Patent application and its processing up to grant / refusal, maintenance etc. is explained in the following chapters. This manual does
    2. not cover the infringement proceedings. Certain matters like exceptions to the rights of patentee, Government use, Compulsory Licensing etc. have also been included.
  5. Some of the key definitions have been explained in Chapter-2.
  6. Chapters 3 to 7 deal with the procedure of filing a patent application, including Ordinary Application, PCT International Application, PCT National Phase Application, Convention Application, Patent of Addition & Divisional Application.
  7. Chapter 8 & 9 relates to examination, grant, pre-grant and post grant oppositions.
  8. Chapters 10 to 14 cover the post-grant procedures such as maintenance of patent, appeal, revocation, compulsory licensing and use of patents for the purposes of Government.
  9. Chapters 15 to 20 relate to Patent Agents, offences and penalties, general powers of Controller, general services and miscellaneous provisions.
  10. Time limits prescribed under the Act and Rules have been specifically dealt with separately in Chapter 21.
  11. The Officers functioning under the Act have been vested with statutory powers. They also have some discretionary powers under the Act which are to be exercised judiciously. As regards the patentability of any subject matter under consideration by an Officer empowered under the Act, he shall not be guided solely by the contents of this manual but shall take judicious decisions based on the Act, Rules and any judicial decisions on the matter. However, all officers of Patent Office shall follow the procedure set forth in the manual.

Chapter -2 Key definitions

02.01 General
For better appreciation of this manual, the user may require to have clear understanding of certain terms which are defined in the Act or Rules. Some of the important definitions are as under.
02.02 Definitions
02.02.01 "Controller" means the Controller General of Patents, Designs and Trade Marks referred to in Section 73. Controller shall be construed as including a reference to any officer discharging the functions of Controller in pursuance of Section 73. The office of Controller General of Patents, Designs and Trademarks is at Mumbai. The Controller General of Patents, Designs and Trademarks is appointed by the Government of India under Section 3 of the Trademarks Act, 1999. The person so appointed will be the Controller of Patents for the purposes of Patents Act, 1970. The Central Government may also appoint other officers who shall discharge, under the superintendence and directions of the Controller General of Patents, Designs and Trademarks, such functions of the Controller under this Act as he may authorize in writing, from time to time by general or special order. Section 2(1)(b), 73 Section 3 of Trademarks Act, 1999
The Controller General has authorized the Assistant Controllers, Deputy Controllers, Joint Controllers and Senior Joint Controllers to discharge most of the functions under the Act. The Controller General has the power to withdraw any matter pending before an officer, by an order in writing and for reasons to be recorded therein, and deal with such matter himself either de novo or from the stage it was so withdrawn or transfer the same to another officer who may subject to special directions in the order of transfer, proceed with the matter either de novo or from the stage it was so transferred.
02.02.02 "Government undertaking" means any industrial undertaking carried on – a. by a department of the Government, or b. by a corporation established by a Central, Provincial or State Act, which is owned or controlled by the Government, or c. by a Government company as defined in section 617 of the Companies Act, 1956 (1 of 1956), or d. by an institution wholly or substantially financed by the Government. Section 2(1)(h)
02.02.03 "invention" means a new product or process involving an inventive step and capable of industrial application. Section 2(1)(j)
02.02.04 "inventive step" means a feature of an invention that involves technical advance as compared to the existing knowledge or having economic significance or both and that makes the invention not obvious to a person skilled in the art. Section 2(1)(ja)
02.02.05 "capable of industrial application", in relation to an invention, means that the invention is capable of being made or used in Section 2(1)(ac)
an industry.
02.02.06 "legal representative" means a person who in law represents the estate of a deceased person. Section 2(1)(k)
02.02.07 "assignee" includes an assignee of the assignee and the legal representative of a deceased assignee and references to the assignee of any person include references to the assignee of the legal representative or assignee of that person. Section 2(1)(ab)
02.02.08 "patentee" means the person for the time being entered on the register as the grantee or proprietor of the patent. Section 2(1)(p)
02.02.09 "patent office" means the patent office referred to in section 74. Unlike many other Countries, for the purpose of facilitating the registration of patents, Indian Patent Office functions from four locations viz. Kolkata, Delhi, Chennai and Mumbai. Section 2(1)(r), 2(b), 74
02.02.10 “appropriate office” means the appropriate office of the patent office as specified in rule 4. Rule 2(b), 4
02.02.11 "person" includes the Government. Section 2(1)(s)
02.02.12 "person interested" includes a person engaged in, or in promoting, research in the same field as that to which the invention relates. Section 2(1)(t)
02.02.13 "prescribed" means prescribed by rules made under this Act. Section 2(1)(u)
02.02.14 "prescribed manner" includes the payment of the prescribed fee. Section 2(1)(v)
02.02.15 "true and first inventor" does not include either the first importer of an invention into India, or a person to whom an invention is first communicated from outside India. Section 2(1)(y)

CHAPTER 3

Filing of Patent Application

Application
03.01 Applicant An Application for a Patent for an invention may be made by any of the following persons either alone or jointly with any other person: True and first inventor True and first inventor’s assignee Legal representative of deceased true and first inventor or his/her assignee The term "person" as defined in the Patents Act includes Government. The term “person” as defined in the General Clauses Act 1897 includes any company or association or body of individuals, whether incorporated or not. In the case of a limited partnership, the Application may be in the names of all personally responsible partners. True and first inventor does not include either the first importer of an invention into India or a person to whom an invention is first communicated from outside India. The applicant is required to disclose the name, address and nationality of the true and first inventor. S. 6, 134, 135
Assignee can be a natural person or other than a legal person such as a registered company, a research organization, an educational institute or Government. Assignee includes assignee of an assignee also. Wherever, the inventor(s) is/are not the applicant, a proof of right to apply by way of an endorsement in the Application form (Form 1) or an assignment deed shall be submitted. Legal representative means a person who in law represents the estate of a deceased person. In such a case, the Legal Representative should file death certificate (in original or a certified true copy) along with other appropriate legal instruments as Proof of Right. In case of a convention application, the legal representative or assignee of the applicant in the Convention country can also file a Patent Application in India.
03.01.01 Procedure to be followed in case of death of applicant or in case the legal entity ceases to exist If the applicant dies before the grant of patent, a request may be made by a person who would, by virtue of an assignment or agreement made in writing, or operation of law, be entitled to an interest in the patent. This procedure is also applicable to a legal entity, which ceased to exist before the grant of patent, as well to joint applicants where one of the applicants dies. In all these cases, when a request is made in Form-6, the Controller may allow such substitution. However, in case of joint applicants the substitution can only be made with the consent of all the Section 20
other joint applicants’. When there is a dispute between the joint applicants, regarding such substitution, after giving opportunity to all the applicants, the Controller may give such directions as he thinks fit for enabling the application to proceed with. Accordingly, the Controller may direct that the application shall proceed in the name of one or more of the parties alone. Such directions may also relate to the manner in which the application should proceed. Further, the Controller shall not issue any such direction unless: i. the invention is identified in the agreement or assignment by reference to the number of application for the patent, or ii. an acknowledgement, indicating that the assignment or agreement relates to the invention in respect of which the application is made, is produced before the Controller, or iii. the rights of the claimant in respect of the invention have been finally established by decision of a court.
03.02 Jurisdiction Unlike many other Countries, for the purpose of facilitating the registration of patents, Indian Patent Office functions from four locations viz. Kolkata, Delhi, Chennai and Mumbai. Application for Patent shall be filed with the Patent Office Section 74, Rule 4
having the appropriate jurisdiction. Territorial jurisdiction of a patent office is decided based on the following: i) Place of residence, domicile or business of the applicant (first mentioned applicant in the case of joint applicants). ii) Place from where the invention actually originated. iii) Address for service in India given by the applicant, when the Applicant has no place of business or domicile in India. Territorial jurisdictions are presented below: Patent Office Territorial Jurisdiction Mumbai The States of Gujarat, Maharashtra, Madhya Pradesh, Goa, Chhattisgarh, the Union Territories of Daman & Diu and Dadra & Nagar Haveli. Delhi The States of Haryana, Himachal Pradesh, Jammu and Kashmir, Punjab, Rajasthan, Uttar Pradesh, Uttarakhand, National Capital Territory of Delhi and the Union Territory of Chandigarh. Chennai The States of Andhra Pradesh, Karnataka, Kerala, Tamil Nadu and the Union Territories of Pondicherry and Lakshadweep. Kolkata Rest of India (States of Bihar, Orissa, West Bengal, Sikkim, Assam, Meghalaya, Manipur, Tripura, Nagaland, Arunachal Pradesh and Union Territory on Andaman and Nicobar Islands )
When a patent application is filed with an appropriate office, it shall be processed by that office ordinarily. The appropriate office for filing a divisional patent application is the office where the main application is filed, as a divisional application needs to be examined in conjunction with its main application. ( for Divisional Applications, please see Chapter 6) Section 16
A foreign applicant is required to give an address for service in India and the jurisdiction will be accordingly decided. Appropriate office in relation to International Applications under Patent Co-operation Treaty and the National Phase Applications (for the PCT Applications and National Phase Applications, please see Chapter 7) : (1) The receiving office, designated office and elected office for the purposes of International Applications filed under the Treaty shall be the appropriate office in accordance with Rule 4. (2) An International Application under the PCT shall be filed at and processed by the appropriate office in accordance with the provisions of Chapter III of the Patents Rules, 2006, the Treaty and the Regulations established under the PCT. Rule 5 Chapter III of Patents Rules, 2006
03.03 Type of Patent Applications 1. Ordinary Application, i.e., an Application which has been filed directly in the Indian Patent Office. 2. Convention Application, either directly or through S. 7, 54, 135
PCT route (for the Convention Applications, PCT Applications and National Phase Applications, please see Chapter 7). 3. Divisional Application, which can result from division of a Patent Application. 4. Patent of Addition, which may be filed subsequent to the Filing of an Application for Patent for an improvement or modification.
03.04 Filing of a patent application A patent application shall be filed on Form-1 along with Provisional / Complete Specification, with the prescribed fee as given in First Schedule at an appropriate office. However, a provisional specification cannot be filed in case of a Convention Applications (either directly or through PCT routes) (For further description of Provisional/Complete Specifications please see Chapter 5) Normal fee shall be applicable for applications containing upto thirty pages in specification and upto 10 claims. If the specification exceeds thirty pages or claims are more than ten in number, additional fee as given in First Schedule in payable. Section 7 First Schedule
03.04.01 Contents of Patent Application Section 7
A patent application should essentially contain: (for filing of International Application please see Chapter 7) Application for grant of patent in Form-1. A Proof of Right to Rule 8, 13
file the Application is filed either in the Form of Endorsement at the end of the Application Form 1 or by way of separate assignment Form-1
1. Provisional / complete specification in Form-2. 2. Statement and undertaking under Section 8 in Form-3, Form-2
if applicable. An applicant must file Form 3 either along with the application or within 6 months from the date of application 3. Declaration as to inventorship shall be filed in Form 5 for Applications accompanying a Provisional Specification or a Convention Application or a PCT Application designating India. 4. Power of authority, if filed through a Patent Agent. 5. Priority document, if applicable. 6. Every application shall bear the Signature of the applicant or authorized person / Patent Agent along with name and date in the appropriate space provided in the forms. 7. The Specification shall be signed with date on the last page of the Specification. The drawing sheets should bear the signature of an applicant or his agent in the right hand bottom corner. 8. If the Application pertains to a biological material obtained from India, the applicant is required to submit the permission from the National Biodiversity Authority any time before the grant of the patent. However, it would be sufficient if the permission from the National Biodiversity Authority is submitted before the grant of the patent. 9. The Application form shall also indicate clearly the source of geographical origin of any biological material used in the Specification, wherever applicable. Form-3 (Section 8, Rule 12) Form-5 Section 6 of the National Biodiversity Act, 2002
03.04.02 E-filing : 1. The Patent Office provides the facility to file a Patent Rule 6
Application online from the native place of the agent of the applicant or applicant through e-filing. 2. For e-filing, applicant / agent must have a digital signature. For the first time, applicant / agent has to register as new user and has to create login ID and password on the Patent office portal. (http://www.ipindia.nic.in). 3. A preliminary Software (Client Software) has to be downloaded from the above mentioned site and has to be installed on the host computer. With the help of said software, an XML file gets generated and all the relevant documents (i.e. Form 1, Form 2, Form 3, etc.) in soft copy have to be uploaded. An Application number and CBR receipt gets generated after successful uploading. 4. Major objectives and purpose of providing the facilities of e-filing is to save time and other hazards to protect the priority date of Application and time line to enter into National Phase Application, Patent of Addition and Divisional Application within time frame in case of last moment instruction of applicant . 5. The applicant / agent will receive the filing receipt and CBR immediately after acceptance of Application in the software, with Patent Application number, date and time of filing. 6. The Office is in the process of upgrading the e-filing platform so as to enable an applicant to file all subsequent papers electronically. It is also proposed to make e-filing compulsory in the near future.
03.04.02.01 Steps for e-filing of Patent Application 1. For using this Portal click on link 'On-line
Registration for New User' 2. Complete On-line Registration process for getting User ID & Password. 3. Login to e-Patent portal after successful registration. 4. Download Client Software for preparing Patent Application(s) offline. 5. Complete the Patent Application offline and generate an XML file using Client Software. 6. After creating Application (XML) file offline, digitally sign the XML file (Max. file size permitted 15 MB) for uploading to the IPO Server. 7. Login to e-Patent portal for uploading Application XML file on IPO Server. 8. Upload & submit digitally signed XML file to IPO Server. 9. Process the Application for EFT (Electronic Fee Transaction). 10. Review Application Status on e-Patent Portal. 11. On successful EFT, acknowledgement details would be displayed/ generated. 12. Print the Acknowledgement. 13. Detailed user manual in pdf format is uploaded on the official website where Certifying Authority, Authorised Bank, Prerequisites of e-filing, Procedure and guidelines of e-filing of Patent Applications are described in detail. e-filing user manual
03.04.03 Leaving and serving documents at Patent Office 1. Any Application, notice or other document authorised or required to be filed, left, made or given at the Patent office, or to the Controller or to Rule 6
any other person under the Act or these rules, may be tendered by hand or sent by a letter addressed to the Controller at the appropriate Office or to that person through post or registered post or speed post or courier service or by electronic transmission duly authenticated. 2. If it is sent by post or registered post or speed post or courier service or by electronic transmission duly authenticated, it shall be deemed to have been filed, left, made or given at the time when the mail containing the same would have been delivered in the ordinary course of post or registered post or speed post or courier service, or by electronic transmission duly authenticated, as the case may be. In proving such sending, it shall be sufficient to show that the mail was properly addressed and transmitted. 3. In case of a postal or courier delay, the Controller follows the provisions of the above paragraph with regard to the date of receipt of the document. 4. Any written communication addressed to a patentee at his address as it appears on the register of patents or at his address for service given under rule 5, or to any applicant or opponent in any proceedings under the Act or these rules, at the address appearing on the Application or notice of opposition, or given for service, shall be deemed to be properly addressed. 5. All notices and all written communications addressed to a patentee, or to any applicant or opponent in any proceedings under the Act or these rules, and all documents forwarded to the patentee or to the said

applicant or opponent, shall, except when they are sent by special messenger, be sent by registered post or speed post or courier service or by electronic transmission duly authenticated.

  1. The date of a notice or a written communication addressed to a patentee or to any applicant or opponent in any proceedings under the Act and these rules shall be the date of dispatch of the said notice or written communication, by registered post or speed post or courier or fax or electronic transmission duly authenticated, as the case may be, unless otherwise specified under the Act or these rules.
  2. In case of delay in receipt of a document or a communication sent by the Patent office to a party to any proceedings under the Act or these rules, the delay in transmitting or resubmitting a document to the Patent office or doing any act by the party may be condoned by the Controller if a petition for such condoning of delay is made by the party to the Controller immediately after the receipt of the document or a communication along with a statement regarding the circumstances of the fact and evidence in support of the statement:

Provided that the delay condoned by the Controller shall not exceed the period between the date on which the party was supposed to have received the document or communication by ordinary course of mail or electronic transmission and the actual date of receipt of the same.

03.04.04 Receiving documents in Office:

1. The application and any other documents with accompanying fees and/or without accompanying fees is received at the Patent Office at separate counters known as Fee Counter (FC) and Non-Fee Counter (NFC) respectively. 2. Both the counters stand closed at 5 pm for facilitating further processing and no papers will be received after 5 pm. 3. All documents by post/courier are received at a separate counter. The fee bearing documents are sent to the fee counter and the non-fee bearing documents are sent to the non-fee counter. 4. The staff at the fee counter makes relevant entries in the module and generates the Cash Book Receipts (CBRs). The staff at the non-fee counter makes relevant entry in the document receipt module. 5. The staff at the fee counter stamps the documents so received and enters the CBR number, date, amount of fee received, application number, patent number or other relevant entries. The staff at the non-fee counter also stamps the documents after making entries in the module. 6. The documents from both the counters are sent on an hourly basis to the Electronic Data Processing (EDP) Section for digitization. 7. Documents requiring no digitization are sent to the concerned section on daily basis.
03.04.05 Language and Paper size etc. All documents and copies of documents to be furnished shall be written or typewritten or printed either in Hindi or in English language in large and legible characters with Rule 9
deep indelible ink with lines widely spaced upon one side only of strong white paper of a size A4 with a margin of at least 4 centimeters on the top and left hand part and 3 centimeters on the bottom and right hand part thereof. It is desirable that the documents are prepared with lines spacing of 1 1/2 or double space in non-script type font (e.g., Arial, Times Roman, or Courier), preferably in a font size of 12. Signature Any signature which is not legible or which is written in a script other than Hindi or English shall be accompanied by a transcription of the name either in Hindi or in English in block letters (Rule 8).
03.04.06 Sequence listing In case the Application for Patent discloses sequence listing of nucleotides and/or amino acids, the same shall be filed in electronic form. However, the fee with respect to the equivalent number of pages shall be payable. Rule 9
03.04.07 Fee: 1. Fee payable under the Act may either be paid in cash or through electronic means or may be sent by bank draft or cheque payable to the Controller of Patents and drawn on a schedules bank at the place where the appropriate office is situated. If the draft or cheque is sent by post, the fee shall be deemed to have been paid on the date on which the draft or cheque would have reached the Controller in the ordinary course of mail. 2. Where a fee is payable in respect of a document, the entire fee shall accompany the document. 3. Where a fee is payable in respect of the doing of an act by the Controller, the Controller shall not do that act Section 142 Rule 7 First Schedule
until the fee has been paid. 4. In case an application processed by a natural person is fully or partially transferred to a person other than a natural person, the difference, if any, in the scale of fee(s) between the fee(s) charged from a natural person and the fee(s) chargeable from the person other than a natural person in the same matter shall be paid by the new applicant with the request for transfer. 5. Fee once paid in respect of any proceedings shall not be refunded whether the proceedings have taken place or not. 6. Prescribed fee for various proceedings under the Act is given in First Schedule.
03.05 Processing of Application
03.05.01 Initial processing 1. On receipt of an application, the Office accords a date and serial number to it. PCT national phase Applications and non-PCT Applications are identified by separate serial numbers. 2. All applications and other documents are digitized, verified, screened, classified and uploaded to the internal server of the Office. 3. Patent applications and other documents are arranged in a file wrapper and the Bibliographic sheet is prepared and pasted on the file cover, so that the files move on for storing in the compactors. 4. The Application is screened for: a. International Patent Classification. b. Technical field of invention for allocation to an examiner in the respective field. c. Relevance to defence or atomic energy.
d. Correcting/completing the abstract, if required. If found not proper, the abstract will be recasted suitably, so as to provide better information to third parties. 5. Requests for examination are also accorded separate serial number. Section 10(4)
03.05.02 Scrutiny of application
1. The Office checks whether the Application has been filed in appropriate jurisdiction (for jurisdiction please see Chapter 3). If the jurisdiction is not appropriate, the application shall not be taken on record and the applicant is informed accordingly. 2. The Office checks for proof of right to file the application (See Chapter 3). If the proof of right is not filed along with the application, it shall be filed within a period of six months from the date of filing of the application. Otherwise, the applicant shall file the same along with a petition under Rule 137/138. 3. The Office checks whether the application and other documents have been filed in the prescribed format i.e. prescribed forms, request, petitions, assignment deeds, translation etc. Further, the Office checks whether: a) the documents are prepared on a proper sized paper, typed in appropriate font with proper spacing, b) the documents are duly signed. c) abstract, drawings (if any) have been filed in proper format, d) meaningful Claim(s) are present in a complete Specification,
e) Power of Attorney or attested copy of General Power of Attorney (if any) is filed, f) Form-5 has been filed(along with complete after Provisional or for filing PCT-NP/ Convention Application), g) the invention has been assigned to another person and Form 6 has been duly filed. If the right is assigned from an individual to a legal entity, the legal entity is invited to pay the balance fees.
03.05.03 Secrecy Directions and consequences thereof 1. If in the opinion of the Controller an invention pertains to a subject matter relevant for the purpose of defence as notified by the Central Government, the Controller issues a secrecy direction prohibiting the publication of the application to the applicant and refers the matter to the Central Government for their consideration as to whether the application is prejudicial to the defence of India. 2. The Central Government, after considering the merits of the secrecy direction, may give notice to the Controller as to whether the secrecy direction needs to be continued or not. 3. The Central Government reviews the matter at an interval of six months. The applicant may request for a reconsideration of the secrecy direction and if the same is found reasonable by the Controller, he may request the Central Government for a review. 4. If the Central Government is of the opinion that an invention in respect of which the Controller has not imposed a secrecy direction and is relevant for defence Section 35, 36, 37, 38 Notification No. _________
purposes, it may at any time before the grant of the patent notify the Controller to that effect. Thereupon, the Controller invokes the provisions of Section 35(1). 5. So long as any directions under Section 35 are in force, the Controller shall not take a decision on grant/refusal of the application.
03.05.04 Inventions relating to Atomic Energy 1. No Patent is granted in respect of an invention relating to atomic energy falling within sub-section (1) of Section 20 of the Atomic Energy Act, 1962. 2. According to Section 20(1) of Atomic Energy Act, atomic energy means energy released from atomic nuclei as a result of any process including the fission and fusion processes. Under this Act, "prescribed substances" means any substance including any mineral which the Central Government may, by notification, prescribe, being a substance which in its opinion is or may be used for the production or use of atomic energy or research into matters connected therewith and includes uranium, plutonium, thorium, beryllium, deuterium or any of these respective derivative or compounds or any other materials containing any of the aforesaid substances. The Act defines the term "radioactive substances" or "radioactive material" as any substance or material, which spontaneously emits, radiation in excess of the levels prescribed by notification by the Central Government. Prescribed Substances, Prescribed equipment and Technology” have been notified by the Government of India, Department of Atomic Energy vide S.O.61(E), published in the Gazette of India (extraordinary, Part II, Section 3, sub-section (ii), Section 4 of Patents Act. Section 20 of the Atomic Energy Act, 1963. S.O.61(E)
dated 20th January, 2006. 3. Any person desiring to apply for a patent abroad for an invention relating to or which he has reason to believe relates to atomic energy shall obtain prior permission from the Central Government before making the application abroad or communicating the invention to any person abroad, unless six weeks have elapsed since his request for permission was made to the Central Government and no reply was received by him. 4. Upon screening, if an Application is found to be falling within the purview of the Atomic Energy Act, the Controller refers the Application to the Central Government. 5. The Central Government upon consideration may issue a direction to the Controller, which is binding. 6. The opinion of the Central Government is not open to an appeal.
03.06 Withdrawal of patent application: The applicant may, at any time after filing the application but before the grant of a patent, withdraw the application by making a request in writing and by paying the prescribed fee. However, if the applicant makes a request for withdrawal within 15 months from the date of filing or priority of the application, whichever is earlier, the application will not be published. It is desirable that the applicant specifies in the request that such withdrawal is under Section 11A(3)(c). S.11B(4) First Schedule

Chapter- 4 Publication of Application

04.01 Publication of Patent Application (a) An Application for Patent is not open to public before the expiry of 18 months from the date of filing or date of priority, whichever is earlier. (b) At the end of 18 months period from the date of filing or from the date of priority whichever is earlier, the Application is published in the Official Journal except in the cases where: i. Secrecy direction u/s 35 is in force. ii. Application abandoned u/s 9(1) (i.e., complete Specification not filed within twelve months from the date of filing of Provisional Specification). iii. Withdrawn three months prior to the publication period i.e. before the end of 15th month from the date of filing or priority, whichever is earlier. This will apply for National Phase entry of PCT Applications as well. (c) The Patent Office publishes the Application in the Official e-Journal ordinarily within one month from the date of expiry of 18 months from the date of filing or priority, whichever is earlier. (d) In cases, where a secrecy direction has been given, the Application is published, when the secrecy direction is revoked subject to the expiry of the 18-month period. (e) No application will be published unless a power of Section 11A, Rule 24
authority, if applicable, is filed.
04.02 Early Publication (a) A request for early publication may be made in Form-9 with the prescribed fee of Rs.2,500/-for natural person(s) or Rs.10,000 for legal entity other than natural person(s). (b) The request for early publication will be considered if it does not pertain to subject matter relevant for defence or atomic energy. (c) Where a request under (a) above is made, the application is published within one month from the date of such request. Section 11A(2), Rule 24A
04.03 Particulars of Publication The official Patent Office Journal is published on every Friday with the following particulars: i. Application number ii. Date of filing iii. Title of invention iv. Publication date v. International Patent Classification vi. Name and address of the applicant vii. Name of the inventor(s) viii. Priority details like priority document number, date, country etc. ix. Reference to Patent of Addition / Divisional Application along with filing date of the parent Application. x. Abstract xi. No. of claims xii. Drawings (if any) Section 11A
04.04 Effects of Publication 1. Upon publication, the Patent office makes the Specification (complete as well as Provisional, if
Section 11A(6)
any), and drawings filed in respect of the Application available to the public on its website or on payment of the prescribed fee as given in the First Schedule if such a request is filed. 2. After publication of the Application for Patent the depository institution will make the biological material mentioned in the specification, available to the public. 3. A patentee can claim damages from the date of publication of his/her application. However, the patentee can institute a suit for infringement only after a patent is granted. 4. The rights of patentee with respect to applications filed under section 5(2) before 1st day of January, 2005 will accrue from the date of grant of the patent. Further, in such a case, after the grant of a patent, the patent-holder shall only be entitled to receive reasonable royalty from such enterprises which have made significant investment and were producing and marketing the concerned product prior to 1st day of January, 2005 and which continue to manufacture the product covered by the patent on the date of grant of the patent and no infringement proceedings shall be instituted against such enterprises. 5. No patent shall be granted before the expiry of six months from the date of Publication of the Application. Rule 55(1A)

Chapter 05 Provisional and Complete Specification

05.01 Specification
In order to obtain a patent, an applicant must fully and particularly describe the invention therein claimed in a complete specification. The disclosure of the invention in a complete specification must be such that an ordinarily skilled person may be able to perform the invention. This is possible only when an applicant discloses the invention fully and particularly including the best method of performing the invention. The Specification is a techno-legal document containing full scientific details of the invention and claims to the patent rights. The Specification, thus, forms a crucial part of the Patent Application. It is mandatory on the part of an applicant to disclose fully and particularly various features constituting the invention. The Specification may be filed either as a Provisional or as a complete Specification. The Specification (provisional or complete) is to be submitted in Form-2 along with the Application in Form-1 and other documents, in duplicate, along with the prescribed fee as given in the First Schedule. The first page of the Form 2 shall contain: (a) Title of the invention; (b) Name, address and nationality of each of the applicants for the Patent; and (c) Preamble to the description. The title of the Specification shall sufficiently indicate the specific features of the invention. Every Specification whether Provisional or complete shall describe the invention. The Section 9, 10. Rule 13.
applicant shall submit drawings, wherever required. The Controller may also require the applicant to submit drawings, if necessary at the examination stage. Such drawings shall form a part of the Specification and suitable references thereto shall be made in the Specification. The Controller may require the applicant to submit, anytime before the grant, models or samples related to the invention for better illustration of the invention. However, such models or samples shall not form part of the Specification.
05.02 Provisional Specification a) When the applicant finds that his invention has reached a presentable form but not the final shape, he may prepare a disclosure of the invention in the form of a written description and submit it to Patent office as a Provisional Specification which describes the invention. b) A Provisional Specification secures a priority date for the application over any other application which is likely to be filed in respect of the same invention being developed concurrently. c) Immediately on receiving the Provisional Specification the Patent office accords a filing date and application number to the Application. Section 9
d) An application accompanying a provisional Section 9 (1),
specification is deemed to be abandoned if no complete specification is filed within twelve months from the date of filing of the provisional specification. However, the applicant has an option to post-date the provisional specification under Section 17, before the expiry of twelve months from the date of filing. Such request for post-dating shall not be allowed after the expiry of twelve months from the date of application. Such 11A(3)(b)
abandoned applications are not published. Once abandoned for reasons mentioned above, no priority can be claimed from such application, for any other application such as divisional application etc.
e) If two provisional specifications filed by an applicant are cognate or if one is a modification of the other, the applicant may file one complete specification covering both the provisional applications. Such a complete specification shall have to be filed within twelve months from the date of filing of the first provisional application. Section 9(2)
f) An applicant may, within twelve months from the filing of a complete specification (not being a convention application or a PCT National Phase Application), convert the same into a provisional specification. Consequently, the applicant has to file a complete specification within twelve months from the date of first filing. Section 9(3)
g) A provisional specification (i.e. the one filed directly or the one converted from a complete specification) may be post-dated to the date of filing of the complete Specification. Section 9(4)
05.02.01 Provisional Specification – General A Provisional Specification is not a rough draft or a skeleton of the complete Specification. The complete Specification, which follows a Provisional Specification, does not replace the latter. Both are permanent and separate documents and remain so in the file.
05.02.02 Contents of Provisional Specification 1. A Provisional Specification shall essentially contain the title and description of the invention of the invention and shall start with a preamble ‘The following
Specification describes the invention.’ Claims may not be included in the Provisional Specification as the purpose of filing a Provisional Specification is to claim a priority date. 2. The description starts from the second page starting with the field of invention and containing the background of the invention, object of the invention and statement of the invention. 3. It is advisable to include in the Provisional Specification as much information as the applicant has at the time of filing, but in any case the description should be adequate to distinguish the invention from the prior art. 4. It may be noted that a Provisional Specification cannot be filed in case of a Divisional, Convention or a PCT National Phase Application. In such cases, filing a Complete Specification is a mandatory requirement.
05.03 Complete Specification The complete Specification is a techno-legal document which fully and particularly describes the invention and discloses the best method of performing the invention. As the complete specification is an extremely important document in the patent proceedings it is advised that it should be drafted with utmost care without any ambiguity. The important elements of the complete Specification are further discussed below. Section 10
05.03.01 Complete Specification shall: 1. fully and particularly describe the invention and its operation or use and the method by which it is performed; 2. disclose the best method of performing the invention which is known to the applicant for which he is entitled Section 10(4)
to claim protection; 3. end with a claim or set of claims defining the scope of the invention for which the protection is claimed; 4. make reference to deposit of the biological material in the international depository authority, if applicable.
05.03.02 National phase applications In case of national phase applications the title, description, drawings, abstract and claims filed with the international application are taken as a complete specification. (For further details of the PCT National Phase Applications, please see Chapter 7) Section 10(4A)
05.03.03 Title The title should be sufficiently indicative of the inventive features of the invention. It need not be the same as the preamble of the main claim. It shall be brief, free from fancy expressions or ambiguity and as precise and definite as possible, but it need not go into the details of the invention itself and should normally consist of fifteen words. The following are not permissible in the title: Inventor’s name, the word ‘Patent’, words in other languages, the abbreviation “etc”, fancy words e.g. “Washwell Soap”, “Universal Rest Easy Patent Chair”. R.13(7)(a)
05.03.04 Field of the Invention and use of Invention The description should preferably begin with a general statement of the invention so as to indicate briefly the subject matter to which the invention relates, e.g. “This invention relates to …………………”. Advantages of the invention may be mentioned to bring out clearly the areas of application and preferable use of the invention. The applicant may substantiate industrial applicability of the invention in this part.
05.03.05 Prior Art and problem to be solved This part should indicate the status of the technology in the
field of invention with reference to developments in the field, patents and pending patent applications in the specific art. When the invention relates to an improvement on an existing product or process, a short statement of the closest prior art known to the applicant shall also be given. However, the description should fully and particularly describe the invention, by clearly distinguishing it from such a closest prior art, known to the applicant.
05.03.06 Objects of the Invention The purpose of this part is to clearly bring out the necessity of the invention. It shall clearly mention the technical problems associated with the existing technology and the solution for that, bringing out the obvious differences between the claimed invention and the prior art. The solution sought by the invention should be clearly brought out as object(s) of inventions with statements like “It has already been proposed ………………” followed by the objects which the inventions has in view e.g. “The principal object of this invention is ……………”, “Another object of this invention is ……………..”, “A further object of this invention is ………….”etc.
05.03.07 Summary of the Invention The description should include a summary of invention before giving the details of the invention and the method of performing it. The statement should clearly set forth the distinguishing novel features of the invention for which protection is desired. This part is intended to declare different aspects of the invention.
05.03.08 Detailed Description of Invention (with reference to drawings, if any) 1. Description of an invention is required to be furnished in sufficient detail so as to give a

complete picture of the invention and follows the Summary of invention. The nature of improvements or modifications effected with respect to the prior art should be clearly and sufficiently described. The details of invention described should be sufficient for a person skilled in the art to perform the invention. It shall include examples / drawings or both for clearly describing and ascertaining the nature of invention. Sufficient number of examples must be included in the description, especially in the case of chemical related inventions.

  1. The details of invention described should enable a person skilled in the art to reduce the invention into practice without further experimentation.
  2. Reference to the drawings should be specific and preferably in the following form: “This invention is illustrated in the accompanying drawings, throughout which, like reference numerals indicate corresponding parts in the various figures. The Specification in respect of a Patent of Addition should contain at the beginning of the description, a definite statement indicating an improvement in or modification of, the original invention, and the serial number of the Application for Patent in respect of the original invention. The Specifications should also contain a short statement of the invention as disclosed in the earlier Specification.
  3. Terms in other languages, if any, used in the description should be accompanied by their English equivalents. The use of vague slang,
words and colloquialisms is objectionable and shall be avoided. 5. When a biological material is described in the specification and when such material is not available to the public and cannot be described adequately as per the provisions of the Act, such material shall be deposited in order to make the application complete. The deposit shall be made with the International Depository Authority under the Budapest Treaty, on or before the date of filing. The International Depository Authority in India is Microbial Type Culture Collection and Gene Bank (MTCC) – Chandigarh. http://ipindia.nic.in/ipr/patent/d_inst_456.pdf. For further information on Microbial Type Culture Collection and Gene Bank (MTCC) please visit – http://wdcm.nig.ac.jp/CCINFO/CCINFO.xml?773]; http://www.imtech.res.in/mtcc.] 6. Reference to such biological material shall be made in the Specification within three months from the date of filing, giving all the available characteristics of the material required for it to be correctly identified or indicated including the name, address of the depository institution and the date and number of the deposit of the material at the institution. 7. Further, the source and geographical origin of the biological material specified in the Specification shall also be disclosed. 8. In the case of Biotechnology related inventions, relevant numbers of the sequence listing shall be mentioned at appropriate place in the
specification. 9. Sequence listing should also be given in electronic form. However, the fees with respect to the corresponding number of pages should be paid. 10. Access to the material is available in the depository institution only after the date of the application of patent in India.
05.03.09 Drawings 1. Drawings or sketches, which require a special illustration of the invention, shall not appear in the description itself. Such drawings shall be on separate sheet(s). 2. Drawings shall be prepared neatly and clearly on durable paper sheet. 3. Drawings shall be on standard A4 size sheets with a clear margin of at least 4 cm on the top and left hand and 3cm at the bottom and right hand of every sheet. 4. Drawings shall be on a scale sufficiently large to show the inventions clearly and dimensions shall not be marked on the drawings. 5. Drawings shall be sequentially or systematically numbered and shall bear— a. in the left hand top corner, the name of the applicant; b. in the right hand top corner, the number of the sheets of drawings, and the consecutive number of each sheet; and c. in the right hand bottom corner, the signature of the applicant or his agent. 6. No descriptive matter shall appear on the drawings except in the flow diagrams. Rule 15
05.03.10 Abstract
a. Every complete specification shall be accompanied by an abstract to provide technical information on the invention. The abstract shall commence with the title of the invention. b. The abstract shall be so drafted that it constitutes an efficient instrument for the purposes of searching in the particular technical field, in particular by making it possible to assess whether there is a need to consult the specification itself. c. The abstract shall contain a concise summary of the matter contained in the specification. The summary shall indicate clearly the technical field to which the invention belongs, technical problem to which the invention relates and the solution to the problem through the invention and principal use or uses of the invention. Where necessary, the abstract shall contain the chemical formula, which characterizes the invention. d. The abstract may not contain more than one hundred and fifty words. If the specification contains any drawing, the applicant shall indicate on the abstract the figure, or exceptionally, the figures of the drawings which may accompany the abstract when published. Each main feature mentioned in the abstract and illustrated by a drawing shall be followed by the reference sign used in that drawing. e. The Controller may amend the abstract for providing better information to third parties.
05.03.11 Best Method The Act specifically requires that the complete specification must describe the best method of performing the invention known to the applicant, including that, which he may have acquired during the period of provisional protection prior to the date of filing the complete specification. Section 10(4)(a) and 10(4)(b)
05.03.12 Claims Claims define the contours of rights, if and when a patent is granted for an invention. Hence, claims are the most critical part of a Patent Application. In a complete specification the description is followed by claims. Since, claims define the scope of legal protection, it is suggested that they should be drafted carefully to cover all the aspects of the protection being sought at the same time adequately distinguishing the prior art from the claimed invention. Section 10(4)(c)
05.03.13 Unity of invention and clarity of claims 1. Claim(s) of a Complete Specification shall relate to a single invention, or to a group of inventions linked so as to form a single inventive concept. 2. Claims shall be clear and succinct and fairly based on the matter disclosed in the specification. Section 10(5)
05.03.14 Significance of Claims 1. A claim is a statement of technical facts expressed in legal terms defining the scope of the invention sought to be protected. No monopoly is obtained for any matter described in the Complete Specification unless it is claimed in the claims. 2. What is not claimed in the ‘claims’ stands disclaimed, and is open to public use, even if the matter is disclosed in the description. Claims define the boundaries of legal protection sought by the patentee and form a protective fence around the invention which is defined by the words and phrases in the claims. 3. The claims shall define clearly the scope of the invention with conciseness, precision and accuracy, so that others may know the exact boundary into which they should not trespass.
4. Each claim is evaluated on its own merit and, therefore, if one of the claims is objected, it does not mean that the rest of the claims are invalid. It is therefore important to make claims on all aspects of the invention to ensure that the applicant gets the widest possible protection.
05.03.15 Scope of claims 1. Claims must not be too broad to embrace more than what the applicant has in fact invented. A Claim which is too broad may encroach upon the subject matter which is in public domain or belongs to others. 2. However, a claim must not be too narrow also because such a Claim would not be sufficiently effective against potential infringement. An infringer would go scot-free, if the claims were too narrow and hence, the full benefit of the invention may not accrue to the inventor. 3. Having many claims, where each claim has a different scope, allows the applicant to have a legal title to different aspects of the invention. A good drafting may begin with broad claims and develops towards claims that are narrower in scope. 4. Terms of the claim which confuse the scope of the invention, or claims that are not specific (e.g. Any novel matter) should be avoided. 5. A claim shall be for the protection of either a product or process, as the case may be, and shall be in one sentence according to the standard practice.
05.03.16 Structure of Claims 1. The description of invention in the complete specification is to be followed by a “statement of Claims” preceded by the prescribed preamble, “I / We claim” as the case may be. 2. Claims should start from a fresh page, after detailed
description of the invention and should be serially numbered. 3. Each claim should be in a single sentence and should be clearly worded. 4. A claim should not be verbose. 5. There is no restriction as to the number of claims that can be incorporated in the specification. Applicant has to pay additional fee, if total number of claims are more than ten. 6. Each claim should be fairly based on matter disclosed in the specification. This means that all the characteristics of the invention that form part of the Claims must be fully explained in the description. 7. A claim should be clear in the sense that it should not cause the reader to speculate. For example, if words like “thin”, “strong”, “a major part”, “such as”, “when required” or “any” are used, the reader may make a subjective judgment unless such expression follows some definite value. 8. A claim must be specific and not vague, ambiguous or hypothetical in nature. Each claim should be complete so that it covers the inventive feature and enough elements around it to put the invention in the proper context. 9. In Addition, any term, which is used in the claim, must be either found in the description or clearly inferred from the description. 10. Trade Marks are an indication of the origin rather than the composition or content of goods, and should not be used in Patent Applications where a generic term can be used instead. Trade Marks are

only permitted in Claims where it can be shown that their use is unavoidable and does not introduce ambiguity . Where Marks that are registered are mentioned, they should be acknowledged as such. If a Trade Mark is not registered, its owner should be indicated.

11. A Claim usually consists of three parts :

-Preamble,

-Transitional phrase; and

-Body.

  1. The introductory phrase identifies the category of the invention and sometimes the purpose (for example, a machine for waxing paper, a composition for fertilizing soil).
    1. The transition phrase may be words and phrases such as:
    2. - comprising of
    3. -including -consisting of -consisting essentially of
  2. If the invention is an improvement on a product or a process existing in the prior art, the invention should be distinguished very clearly by characterizing the claim with respect to the prior art. In such cases, the claim will have two parts separated by the word ‘characterized by’ or ‘wherein’. The part coming before ‘characterized by’ is the prior art while that comes after will be the features of the invention. In the following example “A data input device” is the introductory phrase, “comprising” is the transition phrase, and the rest of the claim is the body:

“A data input device comprising: an input surface adapted to be locally exposed to a pressure or pressure force, a sensor means disposed below the input surface for detecting the position of the pressure or pressure force on the input surface and for outputting an output signal representing said position and; an evaluating means for evaluating the output signal of the sensor means.”

  1. The first claim is always an independent claim also known as ‘Principal Claim’. It should clearly define the essential novel features of the most preferred embodiment of the process/product that constitutes the invention. The claim may be properly characterized with respect to the ‘prior art’, defining all the technical features essential to the invention or inventive concept. The claim should bring out sufficient details of interrelationship, operation or utility to establish that the invention achieves the intended objectives.
  2. There may be more than one independent claim. There may be more than one independent claim in a single application if the claims fall under a single inventive concept. While there is no restriction as to the number of claims, including independent claims, it is advisable to limit the number of claims, as well as the number of independent claims in a single application so that the claims are all of cognate character and are linked so as to form a single inventive concept. Inclusion of multiple independent claims directed at non-cognate aspects of the claimed invention is not desirable. If claims relate to a plurality of distinct inventions, it may be objected on ground of lack of unity of

invention.

17. A dependent claim derives antecedence from an independent claim and reads into it the features of the independent claim and may contain additional non-essential features and even the minute aspects and optional features. As for example: “A wrapper as claimed in Claim 1, wherein a narrow area of the tear tape, spaced from each edge of the tear-tape, is united to a narrow area of the wrapper defined on each side by a line of perforations which are covered by the outer portions of the tear-tape, the perforations facilitating tearing of the wrapper to remove the portion bounded to the tear-tape.” “A gramophone record according to Claim 1, wherein the percentage of filler employed in the record is from 1 to 70 per cent.” “A tool according to Claim 1, wherein the means for guiding the tool and facilitating the removal of the waste metal and the means for preventing the distortion of the spindle comprise two separate plates slidable and removable mounted on the spindle”.

  1. Further independent claims are justified where the single inventive concept covers more than one category e.g. process, product, complementary versions within one category e.g. plug and socket, transmitter and receiver, which work only together.
  2. Claims should not contain:

1. A claim which is unsearchable by reason of the number of alternatives embraced, or the

choice of characterizing parameters. 2. Dependent Claims that are not fully limited by the terms of the preceding independent claim, e.g. dependent claims which omit, modify or substitute a feature of an independent claim. 20. The practice of including an omnibus claim does not have any legal basis under the Patent Act. In fact, such a claim cannot be allowed as per Section 10(4)(c) of the Act. As such claims are unclear, vague and unsearchable, it is desirable to avoid omnibus claims in a patent application.
05.04 Priority of a claim Each claim of a complete specification shall have a priority date. If the complete specification is filed along with the application, the date of filing of the application is the date of the priority of the claim(s) of the specification if the claim(s) are fairly based on the matter disclosed in the specification. Section 2(1)(w), Section 11 (1), 11 (6)
05.04.01 General Section 11(2),
1) When a Complete Specification is filed pursuant to a Provisional Specification, the priority date thereof shall be the date of filing of the provisional application. 2) When a complete specification is filed based on two or more provisional specifications which are cognate, the priority dates for claims arising from each of the provisional specifications will be the date of the respective provisional specification. 11(3)
3) When a complete specification is converted into a provisional specification and a fresh complete specification is filed thereafter, the priority date of claims shall be the date of disclosure. Section 11(2)
4) When a complete specification is filed after the provisional specification, the Controller may on the request of the applicant made any time before the grant, cancel the provisional specification and postdate the provisional specification to the date of complete specification. 5) If the Claim is fairly based on the matter disclosed partly in one and partly in another such previous application accompanying provisional specification, the priority date of the claim shall be the date of the later filed Specification.
6) Where a complete specification based on a previously filed application in India has been filed within twelve months from the date of that application and the claim is fairly based on the matter disclosed in the previously filed application, the priority date of that claim shall be the date of the previously filed application in which the matter was first disclosed. Section 11(3A)
7) A claim in a complete specification of a patent shall not be invalidated by the reason only of: a. the publication or use of the invention so far as claimed in that claim on or after the priority date of such claim; or b. the grant of another patent which claims the invention, so far as claimed in the first mentioned claim, in a claim of the same or a later priority date. Section 11(8)
05.04.02 Divisional application In case of a Divisional Application, priority date of the claim(s) is the date of filing of the first mentioned application. Section 11(4)
05.04.03 Convention application The priority date of the claim(s) of a convention application is the date of filing of the claim(s) in the convention country Section 11(6), 137
05.04.04 Effect of Priority Date of a Claim The novelty of a claim is dependant on its date of priority. Nothing published on or after the date of priority of a claim can be cited to destroy the novelty of that invention. Section 11(8)

Chapter 6 Divisional Application and Patent of Addition

06.01 Divisional Application
06.01.01 General When a complete specification contains more than one invention, the applicant on his own or to meet the office objection, may divide the application and file further application(s) in respect of inventions disclosed in the provisional or complete specification already filed. Such division shall only be based on plurality of invention in the first mentioned specification. Examination of a divisional application shall always be done in conjunction with the main application. If two or more divisional applications are filed based on a main application, examination of the second or subsequent divisional application shall be done in conjunction with the main application, and the other divisional application(s), examined earlier, to avoid double patenting. The whole patent family, in such cases, is available to the Examiner on the Official database. Maintainability of a divisional application shall be decided by the Controller after examination. The date of filing of a divisional application shall be the same as that of the main application, from which it has been divided. The term of patent for a divisional application shall be twenty years from the date of filing of main application or international filing date in case the application was divided out of National phase application under PCT. Section 16(1)
06.01.02 Contents The Divisional Application should be filed along with a Complete Specification. The Claims of the Divisional Application shall not conflict with those of main application. Section 16(3)
06.01.03 Priority The Divisional Application is treated as a substantive Application and is accorded the date of filing of the main Application. The Claims of Divisional Application shall have the same priority date as that of the main application. A divisional application shall be examined in conjunction with the first mentioned application (main or parent) so as to avoid claim overlap resulting in double patenting. While a patent application can be divided into multiple divisional applications at one instance, a divisional application cannot be further divided. Section 16, Explanation Section 11(4)
06.01.04 Appropriate office A divisional application is examined with reference to the parent application. Therefore, a divisional application shall be filed in the same jurisdiction as that of the main application. The divisional application shall be examined in conjunction with the main application. The change of agent shall not affect the jurisdiction. Rule 4
06.01.05 Reference The Complete Specification accompanying the Divisional Application shall contain a specific reference to the main application number. R.13(2)
06.01.06 Fee For all purposes under the Patents Act, the Divisional Application is treated as a substantive application and, hence all fee applicable to a patent application shall be payable. S.16
06.02 Patent Of Addition
06.02.01 General a) When an applicant comes up with an improvement or modification of the invention described or disclosed in main application for which he has already applied for or has obtained a patent, the applicant may make an application for patent of Section 54, 55
Addition. b) An Application for a Patent of Addition shall be filed on the same or subsequent date of filing of the Application for main Patent. c) A Patent of Addition shall be granted only after the grant of the main patent. d) When a patentee holds two patents, it is possible to convert one of the independent patents to a patent of addition of the other, if the subject matter was an improvement in or modification to the other patented invention. e) There is no need to pay separate renewal fee for the Patent of Addition during the term of the main patent. f) A patent of addition expires along with the main patent. However, if the main patent is revoked, the patent of addition may be converted into an independent patent, if so requested by the patentee. g) Date of filing shall be the date on which the application for patent of addition has been filed.
06.02.02 Novelty and inventive step An application for Patent of Addition cannot be challenged on the ground of lack of inventive step with respect to the disclosure in the main application or patent. But the disclosure in main application or patent may be cited for novelty against the application for patent of addition. Section 56
06.02.03 Reference The complete specification of application for patent of addition shall include specific reference to the number of main patent or the application for main patent, as the case may be, and a definite statement that the invention comprises an improvement in, or a modification of the invention claimed in the specification of the main patent, granted or applied for. Rule 13(3)

Chapter 07 Convention Application, International Application and National Phase Application

07.01 Convention Application
07.01.01 Paris Convention and WTO Agreement 1. This part is directed to applications for patents that enter India through Paris Convention as well as PCT route. 2. Paris Convention of 1883 provides reciprocity in filing with right of priority. India became member of Paris Convention in 1998. 3. India is also a member of WTO Agreement (1995). 4. India is also a member of Budapest Treaty on the International recognition of the deposit of micro-organism for the purpose of patent procedure (1977) and provides a mechanism for depositing biological material in the internationally recognized depository authorities for the purposes of supplementing the description of a Specification 5. India is also a member of Patent Co-operation Treaty. Section 133
07.01.02 Convention Country Any country, which is a signatory or party or a group of countries, union of countries or inter-governmental organizations which are signatories or parties to an international, regional or bi-lateral treaty, convention or arrangement to which India is also a signatory or party and which affords to the applicants for patents in India or to citizens of India similar privileges as are granted to their own citizens or citizens to their member countries in respect of the grant of patents and protection of patent rights shall be a convention country or convention countries for the purposes of this Act. At present, as India is a member of WTO and a member country in the Paris Convention and a contracting state to the PCT, any country, union Section 2(1)(d), 133, 134 List of Convention
of countries or inter-governmental organizations which are members/contracting states to the above convention/treaty/agreement are convention countries for the purposes of the Act. Countries
07.01.03 Convention Application a) Where a person has made an Application for a Patent in respect of an invention in a Convention country (basic application), and that person or the legal representative or assignee of that person makes an application under this Act for a patent within twelve months after the date on which the basic application was made, the priority date of a claim of the complete specification, being a claim based on matter disclosed in the basic application, is the date of making of the basic application. Where applications have been made for similar protection in respect of an invention in two or more convention countries, the period of twelve months referred to in this sub-section shall be reckoned from the date on which the earlier or earliest of the said applications was made. b) Where applications for protection have been made in one or more convention countries in respect of two or more inventions which are cognate or of which one is a modification of another, a single convention application may be made in respect of those inventions at any time within twelve months from the date of the earliest of the said applications for protection. However, the fee payable in respect of such application shall be the same as if separate applications have been made in respect of each of the said inventions. Section 135
07.01.04 Documents to be submitted Every convention application shall 1. be accompanied by a complete specification; 2. an abstract; 3. specify the date on which and the convention country in which, the application for protection, or as the case may be, the first of Section 136
such applications was made; and 4. state that no application for protection in respect of the invention had been made in a convention country before that date by the applicant or by any person from whom he derives title. 5. a complete specification filed with a convention application may include claims in respect of developments or, additions to, the invention in respect of which the application for protection was made in a Convention country. 6. If the Controller requires, a certified copy of the priority document has to be filed within 3 months from the date of communication by the Controller, of such requirement. If the priority document is in a language other than English, an English translation shall be submitted. 7. Where the applicant has complied with Rule 17.1(a), (b) or (bbis), the International Bureau shall, at the specific request of the designated Office, promptly but not prior to the international publication of the international application, furnish a copy of the priority document to that Office. Section 138, Rule 121
07.01.05 Multiple priorities a) When two or more applications for patents constituting one invention have been made in one or more convention countries, one application may be made within twelve months from the date on which the earlier or earliest of those applications was made. b) The priority date of a claim is the date on which the matter was first disclosed in a patent application. Section 135(2), 137
07.01.06 Other conditions a) The term of patent of a convention application shall be twenty years from the date of filing of the Application in India. b) A Convention Application can be divided, and the divided Application shall have the same priority date. c) A Convention Application shall not be post-dated under sub- Section 136, 139
section (1) of section 17 to a date later than the date on which under the provisions of this Act the application could have been made.
07.02 International Application under PCT
07.02.01 PCT International application by Indian applicant An Indian applicant can file a PCT International application in the following manner: a) Indian Patent Office as receiving office after taking permission u/s 39 from the Indian Patent Office. b) Filing directly in the International Bureau of WIPO after taking permission u/s 39 from the Indian Patent Office. c) After filing a patent application in India, anytime before the expiry of 12 months from the date of filing, file an international in IB of WIPO, after taking permission u/s 39 from the Indian Patent Office if the international filing is within 6 weeks from the date of filing in India.
07.02.02 Indian Patent Office as receiving office a) An international patent application can be filed in Indian Patent Office as a receiving office, in request form (PCT / RO / 101) in triplicate, which can be obtained free of charge from the Receiving Office or from the International Bureau of WIPO or can be downloaded from WIPO website. b) In this case, the applicant shall make request for permission under Section 39. After the grant of such permission, the Indian Patent Office shall transmit the patent application to the IB. Applicant has to pay the prescribed transmittal fee in addition to the International Application fee and search fee. International Bureau as receiving office An international patent application can be directly filed in IB along with the prescribed fee, in request form (PCT / RO / 101) in duplicate. However, for filing directly in IB permission u/s 39 is required. Section 2 (1)(a), 39 Section 7(1A)
After filing a patent application in India a) An international patent application can be filed within twelve months from the filing patent application in India, in the Indian Patent Office, as a receiving office. However, if such an application is filed before the expiry of 6 weeks from the date of filing in India, permission u/s 39 is required. Applicant has to pay the prescribed transmittal fee in addition to the International Application fee. b) An applicant has another option to file an International Application within twelve months from the filing patent application in India, directly in IB, claiming the priority of the previously filed Indian Patent Application along with the prescribed application fee. A certified copy of the Indian Patent Application may be filed with the International application or within sixteen months from the date of priority.
07.02.03 Requirements of PCT International Application a) The International Application should contain a request, a description, one or more claims, one or more drawings (where required) and an abstract. It should comply with the prescribed physical requirements, should be in one of the prescribed languages, and the required fee should be paid. b) The request may also be presented as a computer printout as prescribed by Section 102(h) of the PCT Administrative Instructions or, alternatively, as a computer printout prepared using the PCT-EASY software, in which case it must be accompanied by a computer disk containing a copy of the data as contained in the request in electronic form and copy of the abstract. c) The request should contain a petition i.e. a request that the International Application be processed according to the PCT. d) It should further contain the title of the invention with necessary data concerning the applicant, the inventor and the agent

representing the applicant. It must be signed by the applicant or his agent. Declaration of inventorship should be signed by the inventor(s) / the applicants in convention country, as applicable, and not by the agent. Where there are two or more applicants, each applicant should sign at his choice either the request or, if the request is signed by an agent, a separate power of attorney. The request should also contain details of priority (where applicable) and an indication of competent International Searching Authority. Following are the competent International Searching Authorities for an Indian Applicant:

a.
Austrian Patent Office (AT)
b.
Australian Patent Office (AU)
c.
European Patent Office (EP)
d.
China Intellectual Property Office (CN)
e.
United States Patent & Trademark Office (US)
f.
Swedish Patent Office (SE)

e) The request may contain some optional indications, in particular, a priority claim according to the Paris Convention for the Protection of Industrial Property.

f) A certified copy of the priority document is required for each priority claimed in the application and the same is to be furnished within 16 months from the priority date. The copies for the designated offices are prepared by the International Bureau at no additional cost to the applicant.

g) It is important to note that International Application filed under new Section 801 (administrative section of PCT) may only be filed with Receiving Offices, which are prepared to accept them, and on such electronic media specified by the Receiving Offices (for further details pl. See PCT Applicant’s Guide).

h) Language of International Application must be one, which the Receiving Office accepts for that purpose (Rule 12.1(a)). If Application is filed in any Receiving Office in India it has to be

either in English or Hindi. However, an application in Hindi shall be accompanied by a duly verified English translation. i) Neither the Treaty nor the Regulations enumerate the languages in which International Applications may be filed. Whether a given language can be used depends on the readiness of the Receiving Office to accept International Applications in that language. Each Receiving Office must, however, accept at least one language for the filing of International Applications, which is both a language accepted by at least one International Searching Authority, competent for the International searching of International Applications filed with that Receiving Office and one of the language of Publication (i.e. Chinese, English, French, German, Japanese, Spanish, Russian or Arabic). In other words, either the International Application in its original language or the translation will be sufficient for the processing by the Receiving Office, for International search and for International publication. j) The request must always be filed in a language that is accepted by the Receiving Office and which is also one of the eight languages of publication. k) Claims: The Claims must define the subject matter of the invention for which protection is sought. They must be clear and concise and fully supported by the description. l) With respect to the structure and drafting of Claims, the PCT requirements are largely similar to what is accepted in most Patent Offices. m) Drawings: The drawings are only required where they are necessary for the understanding of the invention. This will be the case for example for an engineering type of invention. It will not be the case when an invention cannot be drawn, as is the case for a chemical product. Here again, the requirements are similar to those of most Patent Offices. n) Abstract: The abstract is intended to serve the purpose of technical
information. The Treaty says clearly that it cannot be taken into account for any other purpose. This means in particular that it cannot be used for the purpose of interpreting the scope of the protection sought. o) The abstract consists of a concise summary of the disclosure of the invention as contained in the description, Claims and drawings in preferably within 50 to 150 words. It must be drafted in a way, which allows the clear understanding of the technical problem, the gist of the solution of that problem through the invention, and the principal use of the invention. p) Sequence Listing: PCT allows a Designated Office to require that a copy of a sequence listing be filed only on an electronic medium. q) For the applicants who do not wish to file the sequence listing part of their International Applications under new Section 801 (administrative section of PCT), the current provisions will continue to apply, including the filing in written form only (under Rule 5.2) and the concurrent or subsequent furnishing, as provided under PCT Rule 13 and Section 208, of the sequence listing parts in computer readable form but only for the purposes of International search and / or International preliminary examination. In such cases the current system for calculating the basis fee, on the basis for the total number of sheets of the International Application including the sequence listing part, will continue to apply (see item 1(b) of the Schedule of Fees). r) Guidelines for preparing/drafting an International patent application are provided in PCT Applicant’s Guide. PCT Applicant’s Guide
07.02.04 Functions of Indian Patent Office as Receiving Office
1. Patent Office receives the PCT International Application from the applicant or from his authorized Agent. 2. The Office accords the date of receipt of international application Article 11
as the international filing date, provided that the Office has found that, as the time of receipt: i. The applicant does not obviously lack, for reasons of residence or nationality, the right to file an international application with the Office. ii. The international application is in the prescribed language. iii. The international application contains at least the following elements: -an indication that it is intended as an international of PCT
application, -the designation of at least one contracting state, -the name of applicant, as prescribed, -a part which on the face of it appears to be a description,a part which on the face of it appears to be claim(s). 3. (a) If the Office finds that the international application did not, at the time of receipt, fulfill the requirement listed in para 2 above, the Office, as provided in the regulations, invites the applicant to file the required correction. (b) If the applicant complies with the invitation, as provided in the regulations, the Office accords the date of receipt of the required correction as the international filing date. 4. (a) Then the Office checks whether the International Application contains any of the following defects: i. it is not signed as provided in the regulations, ii. it does not contain the prescribed indications concerning the applicant, iii. it does not contain a title,
iv. it does not contain an abstract, Article 14
v. it does not comply to the extent provided in the regulations with the prescribed physical requirements. (b) If the Office finds any of the above defects, it invites the applicant to correct the international application as soon as of PCT
possible, ordinarily within one month from the date of filing, giving a time limit of two months for correction. If the applicant fails to make correction within the prescribed time limit of two months or the time limit as extended by the Office, the application shall be considered withdrawn and the Office declares so. (c) If the international application refers to drawings which, in fact, are not included in that application, the Office notifies the applicant accordingly and if the applicant furnishes the same within two months or within the time limit as may be extended by the Office, the international filing date shall be the date on which the drawings are received by the Office. Otherwise, any reference to the said drawings shall be considered non-existent. (d) If the Office finds that within the prescribed time limits the fee prescribed under Art.3(4)(iv) has not been paid, or no fee prescribed under Art.4(2) has been paid in respect of any of the designated states, the international application shall be considered to be withdrawn and the Office declares so. (e) If the Office finds that fee prescribed under Art.4(2) has been paid in respect of one or more (but less than all) designated states within the prescribed time limit, the designation of those states in respect of which it has not been paid within the prescribed time limit shall be considered and the Office declares so. (f) If, after having accorded an international filing date to the international application, the Office finds, within the prescribed time limit, that any of the requirement listed in 4(a) above, was not complied with at that date, the application shall be considered as withdrawn and the Office declares so. 5. If the language of filing of the International Application is the one acceptable to the Office but not acceptable by the International Searching Authority to carry out International search, the
applicant is required to furnish, within one month from the filing date of the Application, the translation into a language among the following: -a language accepted by the International Searching Authority to carry out International search; -a language of publication by IB (Arabic, Chinese, English, French, German, Japanese, Korean, Portuguese, Russian or Spanish) 6. In cases, where the applicant fails to furnish, within the applicable time limit, a translation for the purpose of International search, the Office invites the applicant to furnish the missing translation. Where the applicant does not furnish the missing translation within the time limit fixed in the invitation, the International Application will be considered withdrawn and the Office declares so. 7. Not all the requirements of the International Application are required to be examined by the Office. For instance, the Office does not deal with substantive questions such as, whether the disclosure of the invention in the Application is sufficient and whether the requirement of unity of invention is complied with. It also does not check all the many detailed physical requirements of the International Application. 8. Those requirements are only checked to the extent that compliance with such requirements is necessary for the purpose of reasonably informed International publication. 9. Typical examples of defects, which may be corrected without affecting the International filing date, are: -Non-payment or partial payment of fees; -Lack of signature in the request; -Lack of a title of the invention; -Lack of an abstract; -Physical defects.
However, in all such cases, non-correction within the prescribed time limit leads to the Application being considered withdrawn as mentioned in the foregoing paragraphs. 10. All the actions as regards to non compliance of the requirements of the International PCT application are subject to strict observance by the Patent Office of the PCT Receiving Office Guidelines. 11. The Office then transmits the “record copy” of the International Application to the International Bureau and the “search copy” to the International Searching Authority. The Office keeps a third copy, the “home copy”. The transmittals do not take place if, and as long as, national prescriptions concerning national security apply and the provisions of Section 35 follow. 12. The Office then mails the record copy promptly to the International Bureau and in any case not later than five days prior to the expiration of the 13th month from the priority date. 13. The search copy must be transmitted by the Office to the International Searching Authority at the time of transmittal of the record copy to the International Bureau except, where the search fees has not been paid on time, in which case, the transmittal of search copy takes place after that fees has been paid. PCT Receiving Office Guidelines
07.02.05 PCT Fee (may vary from time to time) 1. All PCT fees are subject to change periodically. For latest fees, please refer the latest PCT newsletter at URL www.wipo.int. 2. Transmittal fee: as given in the First Schedule. 3. Failure to pay fees or underpayment of fees can be corrected under PCT rule 16 bis. An invitation to pay missing fees will be issued by the Receiving Office. Payment can be made within a month from International filing date or later with a late payment fee. (For further details see 06.02.03) 4. An Indian applicant, filing an International Application under
Patent Cooperation Treaty, is required to remit the consolidated amount towards International filing fee and search fee in US Dollar by Demand Draft, payable to the Controller of Patents at State Bank of India, New York Branch. Transmittal Fee payable to the Indian Patent Office shall be paid in INR to the Controller of Patents, preferably along with the application but not later than one month from the date of receipt of the International Application.
07.02.06 International Search 1. International Search report is established by the International Searching Authority designated by the application in International Application. 2. If the International Application did not claim priority of a previously filed Indian Patent Application, the International search report is normally available within nine months from the International filing date. If priority is claimed, that report is available usually by the 16th month from the priority date. Even where priority is claimed, the International search report is normally available in time before publication of the International Application. This allows time for the applicant to withdraw the Application before publication, if desired. 3. On receipt of the International Search Report the applicant may amend the Claims (under Article 19) in light of the International Search Report with effect in all designated States. The time limit referred to in Article 19 is two months from the date of transmittal of the international search report to the International Bureau and to the applicant by the International Searching Authority, or 16 months from the priority date, whichever time limit expires later. However, any amendment made under Article 19 which is received by the International Bureau after the expiration of the applicable time limit is considered to have been received by the Bureau on the last day of the time limit if it reaches before the
technical preparations for international publication have been completed. 4. Such amendments save costs for preparation of different sets of amendments and for local Agents filing such amendments before Designated Offices. 5. Indian Patent Office has been recognized as an International Searching Authority and an International Preliminary Examination Authority at the meeting of the General Assemblies of WIPO held in September – October, 2007. 6. Once the Indian Patent Office commences operation as an ISA and an IPEA, the International search and examination reports of the International Applications filed in the Patent Office will be issued by the Office.
07.02.07 Withdrawals in International Application Rule 90bis of PCT Regulations
Withdrawal of Application 1. The applicant may withdraw the international application at any time prior to the expiration of 30 months from the priority date. 2. Withdrawal shall be effective on receipt of a notice addressed by the applicant, at his option, to the International Bureau, to the receiving Office or, where Article 39(1) applies, to the International Preliminary Examining Authority. 3. No international publication of the international application shall be effected if the notice of withdrawal sent by the applicant or transmitted by the receiving Office or the International Preliminary Examining Authority reaches the International Bureau before the technical preparations for international publication have been completed. Withdrawal of Designations 1. The applicant may withdraw the designation of any designated State at any time prior to the expiration of 30 months from the
priority date. Withdrawal of the designation of a State which has been elected shall entail withdrawal of the corresponding election under Rule 90bis.4. 2. Where a State has been designated for the purpose of obtaining both, a national patent and a regional patent, withdrawal of the designation of that State shall be taken to mean withdrawal of only the designation for the purpose of obtaining a national patent, except where otherwise indicated. 3. Withdrawal of the designations of all designated States shall be treated as withdrawal of the international application under Rule 90bis.1. 4. Withdrawal shall be effective on receipt of a notice addressed by the applicant, at his option, to the International Bureau, to the receiving Office or, where Article 39(1) applies, to the International Preliminary Examining Authority. 5. No international publication of the designation shall be effected if the notice of withdrawal sent by the applicant or transmitted by the receiving Office or the International Preliminary Examining Authority reaches the International Bureau before the technical preparations for international publication have been completed. Withdrawal of Priority Claims 1. The applicant may withdraw a priority claim, made in the international application under Article 8(1), at any time prior to the expiration of 30 months from the priority date. 2. Where the international application contains more than one priority claim, the applicant may exercise the right provided for in paragraph (a) in respect of one or more of the priority claims. 3. Withdrawal shall be effective on receipt of a notice addressed by the applicant, at his option, to the International Bureau, to the receiving Office or, where Article 39(1) applies, to the International Preliminary Examining Authority. 4. Where the withdrawal of a priority claim causes a change in the
priority date, any time limit which is computed from the original priority date and which has not already expired shall, subject to paragraph (e), be computed from the priority date resulting from that change. 5. In the case of the time limit referred to in Article 21(2)(a), the International Bureau may nevertheless proceed with the international publication on the basis of the said time limit as computed from the original priority date if the notice of withdrawal sent by the applicant or transmitted by the receiving Office or the International Preliminary Examining Authority reaches the International Bureau after the completion of the technical preparations for international publication. Withdrawal of Supplementary Search Request 1. The applicant may withdraw a supplementary search request at any time prior to the date of transmittal to the applicant and to the International Bureau, under Rule 45bis.8(a), of the supplementary international search report or the declaration that no such report will be established. 2. Withdrawal shall be effective on receipt, within the time limit under paragraph (a), of a notice addressed by the applicant, at his option, to the Authority specified for supplementary search or to the International Bureau, provided that, where the notice does not reach the Authority specified for supplementary search in sufficient time to prevent the transmittal of the report or declaration referred to in paragraph (a), the communication of that report or declaration under Article 20(1), as applicable by virtue of Rule 45bis.8(b), shall nevertheless be effected. Withdrawal of the Demand, or of Elections 1. The applicant may withdraw the demand or any or all elections at any time prior to the expiration of 30 months from the priority date. 2. Withdrawal shall be effective upon receipt of a notice addressed

by the applicant to the International Bureau.

3. If the notice of withdrawal is submitted by the applicant to the International Preliminary Examining Authority, that Authority shall mark the date of receipt on the notice and transmit it promptly to the International Bureau. The notice shall be considered to have been submitted to the International Bureau on the date marked.

Signature

  1. Any notice of withdrawal referred to in Rules 90bis.1 to 90bis.4 shall, subject to paragraph (b), be signed by the applicant or, if there are two or more applicants, by all of them. An applicant who is considered to be the common representative under Rule 90.2(b) shall, subject to paragraph (b), not be entitled to sign such a notice on behalf of the other applicants.
  2. Where two or more applicants file an international application which designates a State whose national law requires that national applications be filed by the inventor and where an applicant for that designated State who is an inventor could not be found or reached after diligent effort, a notice of withdrawal referred to in Rules 90bis.1 to 90bis.4 need not be signed by that applicant (“the applicant concerned”) if it is signed by at least one applicant and
  3. a statement is furnished explaining, to the satisfaction of the receiving Office, the International Bureau, the Authority carrying out the supplementary international search or the International Preliminary Examining Authority, as the case may be, the lack of signature of the applicant concerned, or
  4. in the case of a notice of withdrawal referred to in Rule 90bis.1(b), 90bis.2(d), 90bis.3(c) or 90bis.3bis(b), the applicant concerned did not sign the request but the requirements of Rule 4.15(b) were complied with, or
  5. in the case of a notice of withdrawal referred to in Rule 90bis.4(b), the applicant concerned did not sign the demand but the
requirements of Rule 53.8(b) were complied with. Effect of Withdrawal 1. Withdrawal under Rule 90bis of the international application, any designation, any priority claim, the demand or any election shall have no effect in any designated or elected Office where the processing or examination of the international application has already started under Article 23(2) or Article 40(2). 2. Where the international application is withdrawn under Rule 90bis.1, the international processing of the international application shall be discontinued. 3. Where a supplementary search request is withdrawn under Rule 90bis.3bis, the supplementary international search by the Authority concerned shall be discontinued. 4. Where the demand or all elections are withdrawn under Rule 90bis.4, 5. the processing of the international application by the International Preliminary Examining Authority shall be discontinued.
07.02.08 International Preliminary Examination
Significance 1. International Preliminary Examination is useful in many ways. It is optional for the applicant and provides, in addition to the International Search Report, a second opinion on the usual criteria of patentability before expenses are incurred for the national phase (for translation, fees and foreign Agents etc.). 2. Helps the applicant to adapt the International Application in accordance with the results of the International Search Report; 3. If the report is negative and it is decided to abandon the Application, the applicant can save all the expenses that otherwise would have been incurred. However, the opinions from ISA & IPEA are non-binding on the member countries. International Preliminary Examination Authorities For an Indian Applicant, the following are competent International
Preliminary Examining Authorities (IPEAs): 1. Austrian Patent Office (AT) 2. Australian Patent Office (AU) 3. European Patent Office (EP) (Only if ISA was AT, EP or SE) 4. China Intellectual Property Office (CN) 5. United States Patent & Trademark Office (US) 6. Swedish Patent Office (SE) Demand for International Preliminary Examination 1. The demand for international preliminary examination shall be made separately from the international application. 2. The demand for international preliminary examination may be made to the Indian Patent Office, International Bureau or to any of the six competent International Preliminary Examination Authorities mentioned above. 3. The demand shall contain the prescribed particulars and shall be in the prescribed language and form. 4. The demand shall be subject to the payment of the prescribed fees within the prescribed time limit. 5. The demand for International Preliminary Examination has to be made: a. Within 22 months from the date of priority, or b. Within 3 months from the date of transmittal of International Search Report and written opinion to the applicant or the declaration under Article 17(2), whichever is later. 6. The fees to be paid by the applicant is given in the PCT Newsletter which is available on the WIPO website, www.wipo.int
07.03 PCT National Phase Application
07.03.01 General 1. The national phase follows the international phase. It is necessary for an applicant to file a national phase application in each
designated country, where protection is sought for within the prescribed under PCT i.e. at least 30 months from the priority date. However, this time limit may be increased through National Laws by each member Country. Indian Patent Law provides a time limit of 31 months from the priority date with no scope of extension. Some countries allow extension of such time limit on payment of additional fee. 2. For making a national phase application before a Designated Office, the applicant shall: a. pay the prescribed national fee; and b. file a duly verified translation of the basic application, if necessary. 3. International filing date is the deemed date of filing in India if the applicant enters the national phase in India by filing a National Phase Application within thirty one months from the date of priority. 4. The international filing allows the preservation of priority from the date of filing of first application in the convention country. 5. This is a simple and economical procedure for the applicants seeking protection for their invention in many countries. 6. After the International Application is filed the applicant gets an International Search Report (ISR) and at his option he may get a written opinion on the patentability of the subject matter or an International Preliminary Examination Report (IPER). The applicant, then, may decide to enter a national phase.
07.03.02 Basic Requirements to enter National Phase in India 1. It may be noted that a PCT National Phase Application is a Convention Application under Section 135 of which the filing date is the date of International Application provided the applicant files a National Phase Application. 2. The applicant has to file the National Phase Application within 31 months from the priority date or International Application date, Section 7, 10(4) (A) 135 Rule 20 Rule 20(2),
whichever is earlier. 3. The jurisdiction of filing is the same as that of the ordinary (4)(i), 4(ii)
Application. Address for service in India shall be filed. 4. Where the International Application has not been filed or
published in one of the official languages (Hindi or English), a translation of the application, description, claims (if amended,
both as originally filed and amended together with any statement), drawings, if any, and abstract should be submitted along with the
Application. 5. For National Phase Application, a complete specification is to be
filed with the National Phase Application. For the National Phase Application, the title, the description, drawings, abstract and Section 138(4)
Claims as filed with the International Application shall be taken as the complete Specification. It may be noted that at the time of
filing of National Phase Application, no other amendment shall be allowed in the complete specification as filed in the International
Application. 6. If the applicant makes an amendment for an International Application before ISA and/or IPEA, it shall, if the applicant so
desires, be taken as an amendment before the Patent Office.
7. In case of a change in name of an applicant, if the change has
occurred after the international filing date and has not been reflected in a notification from the International Bureau (Form
PCT / IB/ 306), the change may be effected by filing Form 6 and/or Form 13.
8. If PCT/IB/304 is available on the website of WIPO, the Patent Office shall not require the applicant to submit the priority
document. If not available on the website of WIPO, the Office
may request for the same from the International Bureau.
9. However, if the applicant has not complied with the requirements of rule 17.1 paragraph [a or b] of the regulations made under the Rule 20(4)(ii)
PCT, he shall submit the priority document to the office before the
expiry of thirty one months from the date of priority. 10. Where the applicant does not comply with the requirements of (9) above, the Controller shall invite the applicant to file the priority document or the translation thereof within three months from the date of such invitation. If the applicant fails to do so, the claim of the applicant for the priority shall be disregarded for the purposes of the Act. 11. All other formalities that are required for filing and processing an ordinary patent application shall apply to a National Phase Application. 12. Processing of a national phase Application will not commence before the expiry of 31 months from the date of priority. However, the applicant may file an express request for processing before 31 months, in Form 18.

Chapter 08 Examination & Grant

08.01 Request for Examination
1. An Application for a Patent will not be examined unless the applicant or any other interested person makes a request for examination. The request is to be filed in Form 18 with the fee as prescribed in First Schedule. 2. A request for examination has to be made within forty-eight months from the date of priority of the application or from the date of filing of the application, whichever is earlier. If no such request for examination is filed within the prescribed time limit, the application shall be treated as withdrawn by the applicant. 3. In a case where secrecy direction has been issued under Section 35, the request for examination may be made within six months from the date of revocation of the secrecy direction, or within forty-eight months from the date of filing or priority, which ever is later. 4. The Office will not examine an application unless it is published and a request for examination is filed. 5. When a request for examination is filed by an interested person other than the applicant, the Examination Report is sent to the applicant only, and intimation is given to the interested person. Section 11B. Rule 24B. First Schedule.
08.02 Reference for Examination 1. Once a request for examination is received, and the application is published u/s 11-A, the application is Section 12, Rule 24B(2)(i)
taken up for Examination in the chronological order of filing of request for examination. 2. The patent application is referred to an Examiner by the Controller for conducting the formal as well as substantive examination as per the subject matter of the invention vis-à-vis the area of specialization of the Examiner. At present the Patent Office has four examination groups based on the broad area of specialization viz.: a. Chemistry and allied subjects. b. Biotechnology, Microbiology and allied subjects. c. Electrical, Electronics & related subject d. Mechanical and other subjects. The reference to the Examiner is made ordinarily within one month from the date of publication or one month from the date of request for examination, whichever is later. 3. When an application is referred by the Controller, the Examiner makes a report on the patentabilility as well as other matters.
08.03 Examination of application
08.03.01 Report of Examiner 1. The examiner makes a report to the Controller after carrying out detailed examination and investigation, under Section 12, regarding the patentability of the invention. 2. The examiner prepares the report after conducting a prior art search to ascertain the novelty, and examining as to whether the invention disclosed in the specification is inventive and industrially applicable. The Examiner also examines whether the invention belongs to one of the categories of non-patentable inventions coming under Section 3 and 4, and whether there is any other lawful ground of objection to the grant of patent under the Act. Section 12
08.03.02 Search by the Examiner 1. The examiner conducts a search in the Indian Patent Database and all the patent databases of other countries available through the following to determine the patentability: EPO database, WIPO database, USPTO database, JPO DATABASE, TKDL (Traditional Knowledge Digital Library). In addition to the above, PCT Minimum required databases like STN, QPAT, QWEB, DELPHION and non-patent literature including 107 technical and scientific journals. 2. The examiner records in his noting, the following items: a. International classification. b. Search strategy adopted. c. Key word(s) used. d. Language of search. e. Databases consulted for both Patent and non Section 13
Patent literature. f. List of search statements. g. Final findings. h. Limitation on search if any, such as non clarity of claims or multiplicity of inventions or any other reason due to which a reasonable search cannot be conducted. i. Analysis indicating similarities of the invention with the citations.
08.03.03 Novelty a) An invention is considered as new if it is not anticipated by prior publication, prior use or prior public knowledge. An invention is new (novel) if it has not been disclosed in the prior art, where the prior art means everything that has been published, presented or otherwise disclosed to the public before the date of priority, of the Patent Application. For the purpose of determining novelty, an application for Patent filed at the Indian Patent Office before the date of priority of a later filed application but published after the same is also considered as a prior art. b) While ascertaining novelty, the Examiner takes into consideration, inter alia, the following documents: which have been published before the date of priority of the application. such Indian Patent Applications which have been filed before the date of priority but published after and claims the same subject matter. also the Examiner may consider such documents which have been published before in Section 2(1)(j), 13, 29 to 34
a transaction of a learned society or exhibited before in an authorized manner as designated by the Government within one year from the date of such filing. c) A prior art will be considered as anticipatory if all the essential features of the invention under examination are present in the cited prior art. d) The prior art should teach the invention either in explicit manner. e) The relevant date in determining novelty, i.e., anticipation is the date of priority of the claim. f) Mosaicing of prior art documents is not followed in the determination of novelty. g) A generic disclosure in the prior art does not take away the novelty of a specific disclosure. For instance, a metal spring does not take away the novelty of a copper spring. h) A specific disclosure in the prior art takes away the novelty of a generic disclosure. For instance, a copper spring takes away the novelty of a metal spring.
08.03.04 Inventive step
08.03.04.01 General principle a) Inventive step is a feature of an invention that involves a technical advancement compared to the existing knowledge or having economic significance or both and that makes the invention not obvious to a person skilled in the art. An invention is patentable only if it involves one or more inventive step. In relation to the determination of patentability, an Examiner first conducts an enquiry as to the novelty of the claimed invention and then proceeds to Section 2(1)(j), 2(1)(ja)
conduct an enquiry on whether the claimed invention involves one or more inventive step. b) A person skilled in the art is a notional person who is presumed to be a skilled practitioner aware of what was general common knowledge in the relevant art at the relevant date. He would have access to documents cited in the search report, and have had at his disposal the normal means and capacity for routine work and experimentation.
08.03.04.02 Determination of inventive step a) For determination of inventive step, all or any of the prior art(s) revealed during the search process to perform an enquiry as to whether such prior art(s) teaches the claimed invention, are relied upon. b) Publications existing on the date of priority are considered including the Indian Specifications that are filed before and published after with a prior claiming of the same subject matter should not be taken into consideration. c) Invention as a whole shall be considered. In other words, it is not sufficient to draw the conclusion that a claimed invention is obvious merely because individual parts of the claim taken separately are known or might be found to be obvious. d) If an invention lies merely in verifying the previous predictions, without substantially adding anything for technical advancement or economic significance in the art, the inventive step is lacking. e) For the purpose of establishing obviousness of the invention, citing a mosaic of prior arts is permissible, provided such prior art is enabling.
08.03.05 Industrial Applicability
1. In order for an invention to be patentable, an invention must be capable of industrial application. Industrial Application in relation to patentability means that the invention is capable of being made or used in an industry. 2. The Examiner shall assess if the claimed invention is capable of use in any industry or made using an industrial process. Typically the specification explains the industrial applicability of the disclosed invention in a self-evident manner. Usually industrial applicability is self evident. If it is not, a mere suggestion that the matter would be industrially applicable is not sufficient. A specific utility should be indicated in the specification supported by the disclosure. For example, indicating that a compound may be useful in treating unspecified disorders, or that the compound has “useful biological” properties, would not be sufficient to define a specific utility for the compound. The specific usefulness has to be indicated. S.2(1)(ac)
08.03.06 Inventions not patentable The Patents Act provides certain exclusions to patentability. These statutory exclusions are illustrated in the following paragraphs. However, examples given are mere illustrations and may not be conclusive on the subject. Objective decisions may be taken on case to case basis. Section 3
08.03.06.01 An invention which is frivolous or which claims anything obviously contrary to well established laws is not patentable. Some examples of a frivolous nature and contrary to natural laws are:A machine purporting to produce perpetual motion. Section 3(a)
A machine alleged to be giving output without any input. A machine allegedly giving 100% efficiency.
08.03.06.02 An invention, the primary or intended use or commercial exploitation of which would be contrary to public order or morality or which causes serious prejudice to human, animal or plant life or health or to the environment is not patentable. Some examples are: a. Any device, apparatus or machine or method for committing theft/burglary. b. Any machine or method for counterfeiting of currency notes. c. Any device or method for gambling. d. An invention the use of which can cause serious prejudice to human beings, plants and animals. i) Inventions, the intended use or commercial exploitation of which is found to be injurious to public, animal or plant life or health, such as, a method of adulteration of food. ii) An invention, the primary or intended use of which is likely to violate the well accepted and settled social, cultural, legal norms of morality, e.g. a method for cloning of humans. iii)An invention, the primary or proposed use of which would disturb the public order e.g. a device for housebreaking. iv)Terminator gene technology. Section 3(b)
08.03.06.03 The mere discovery of a scientific principle or the formulation of an abstract theory or discovery of any living thing or non-living substance occurring in nature is not patentable. Section 3(c)

i) There is a difference between discovery and invention. A discovery adds to the amount of human knowledge by disclosing something already existing, which has not been seen before, whereas an invention adds to the human knowledge by creating a new product or process involving a technical advance as compared to the existing knowledge.

ii) A claim for discovery of scientific principle is not considered patentable, but such a principle when used with a process of manufacture resulting in a substance or an article may be patentable.

iii)A scientific theory is a statement about the natural world. These theories themselves are not considered patentable, no matter how radical or revolutionary an insight they may provide, since they do not result in a product or process. However, if the theory leads to practical application in the process of manufacture of an article or substance, it may well be patentable. A claim for formulation of abstract theory is not patentable. For example, the fact that a known material or article is found to have a hitherto unknown property is a discovery and not an invention. But if the discovery leads to the conclusion that the material can be used for making a particular article or in a particular process, then the article or process could be patentable.

iv)Finding out that a particular known material is able to withstand mechanical shock is a discovery and therefore not patentable, but a claim to a railway sleeper made of the material would not fall foul of this exclusion, and would be allowable if it passed

the tests for novelty and inventive step. Similarly, finding of a new substance or micro-organism occurring freely in nature is a discovery and not an invention.
08.03.06.04 The mere discovery of a new form of a known substance which does not result in the enhancement of the known efficacy of that substance or the mere discovery of any new property or new use for a known substance or of the mere use of a known process, machine or apparatus unless such known process results in a new product or employs at least one new reactant is not patentable. Section 3(d) provides an explanatory clause to make it more clear which reads as follows: “Explanation:- For the purposes of this clause, salts, esters, ethers, polymorphs, metabolites, pure form, particle size, isomers, mixtures of isomers, complexes, combinations and other derivatives of known substance shall be considered to be the same substance, unless they differ significantly in properties with regard to efficacy”. According to this provision, the following categories are excluded from patentability: (a) mere discovery of a new form of a known substance which does not result in the enhancement of the known efficacy of that substance; (b) the mere discovery of any new property of a known substance; (c) the mere discovery of new use for a known substance; (d) the mere use of a known process unless such known process results in a new product or employs at least one new reactant; (e) the mere use of a known machine or apparatus. Section 3(d)

The explanation to Section 3(d) further clarifies that the salts, esters, ethers, polymorphs, metabolites, pure form, particle size, isomers, mixtures of isomers, complexes, combinations and other derivatives of (any) known substance may be considered to be the same substance and hence not patentable. It however states that such salts, esters, ethers, polymorphs, metabolites, pure form, particle size, isomers, mixtures of isomers, complexes, combinations and other derivatives of such known substance may be considered as patentable only if they differ significantly in properties with regard to efficacy.

The Examiner on a case to case basis applies the test as to what constitutes such salts, esters, ethers, polymorphs, metabolites, pure form, particle size, isomers, mixtures of isomers, complexes, combinations and other derivatives to differ significantly in properties with regard to efficacy from the known substance. The complete specification may bring out clearly and categorically in the description, as to how the subject matter differs significantly in properties with regard to efficacy from the known substance thereof.

In a recent case in relation to a pharmaceutical substance, the Madras High Court held that efficacy means therapeutic efficacy. It was held that:

going by the meaning for the word “efficacy” and “therapeutic”… what the patent applicant is expected to show is, how effective the new discovery made would be in healing a disease having a good effect on the body? In other words, the patent applicant is definitely aware as to what is the “therapeutic effect” of the drug for which he had already got a patent and what is the difference between the therapeutic effect of the patented

drug and the drug in respect of which patent is asked for.” “Due to the advanced technology in all fields of science, it is possible to show by giving necessary comparative details based on such science that the discovery of a new form of a known substance had resulted in the enhancement of the known efficacy of the original substance and the derivatives so derived will not be the same substance, since the properties of the derivatives differ significantly with regard to efficacy.” (Novartis AG Vs. Union of India, W.P. No. 24760/06).
08.03.06.05 A substance obtained by a mere admixture resulting only in the aggregation of the properties of the components thereof or a process for producing such substance is not patentable. An admixture resulting in synergistic properties is not considered as mere admixture, e.g., a soap, detergent, lubricant and polymer composition etc, and hence may be considered to be patentable. A mere aggregation of features must be distinguished from a combination invention. The existence of a combination invention requires that the relationship between the features or groups of features be one of functional reciprocity or that they show a combinative effect beyond the sum of their individual effects. The features should be functionally linked together which is the actual characteristic of a combination invention. In general all the substances which are produced by mere admixing, or a process of producing such substances should satisfy the requirement of synergistic effect in order to be patentable. Synergistic effect should be clearly brought out Section 3(e)
in the description by way of comparison at the time of filing of the Application. The subsequent submissions regarding synergism can be accepted in a reply to the office action as a further support of synergy. However, such submitted data is not allowed to become a part of the Specification.
08.03.06.06 The mere arrangement or re-arrangement or duplication of known devices each functioning independently of one another in a known way is not patentable. In order to be patentable an improvement on something known before or a combination of different matters already known, should be something more than a mere workshop improvement; and must independently satisfy the test of invention or an 'inventive step'. To be patentable, the improvement or the combination must produce a new result, or a new article or a better or cheaper article than before. A combination of old known integers may be so combined that by their working interrelation they produce a new process or an improved result. Mere collocation of more than one integers or things, not involving the exercise of any inventive faculty, does not qualify for the grant of a patent. (Biswanath Prasad Radhey Shyam Vs. Hindustan Metal Industries (1979) 2 SCC, 511). A new and useful application of an old principle may be good subject-matter. An improvement on something known may also afford subject-matter; so also a different combination of matters already known. A patentable combination is one in which the component elements are so combined as to produce a new result or arrive at an old result in a better or more expeditious or more economical manner. If the result produced by the combination is either a new article or a better or cheaper article than before, the Section 3(f)

combination may afford subject-matter of a patent. (Lallubhai Chakubhai Vs. Chimanlal and Co. (AIR 1936 Bom 99.)

An invention claiming a mere juxtaposition of known devices in which each device functions independently is not considered patentable. Merely placing side-by-side old integers so that each performs its own function independently of the others is not a patentable combination. [As for example: a floor mill provided with sieving means]. However, where the old integers when placed together have some working interrelation, producing a new or improved result, then there could be a patentable subject matter in the working interrelation brought about by the collection of the integers.

When two or more features of an apparatus or device are known, and they are juxtaposed without any interdependence on their functioning, they should be held to have been already known. (Rampratap v. Bhabha Atomic Research Center, 1976 IPLR 28 P. 35), e.g., an umbrella with fan (388/Bom/73), bucket fitted with torch, clock and transistor in a single cabinet. These are not patentable, since they are nothing but mere arrangement and rearrangement of items without having any working interrelationship between them and are functioning independently of each other.

As for instance, in the case of an application for a patent in respect of an apparatus for producing metallic bellows, the hydraulic machine and the roll forming machine disclosed therein were functioning as separate machines independently of each other and as such had no novel feature. Hence, there is no invention when a claim is made on known types of hydraulic forming and roll forming machines

functioning independently of each other. A new combination may be the subject matter of a patent although every part of the combination, per se, is old, for here the new article is not the parts themselves but the assembling and working of the parts, together. The merit of a new combination very much depends upon the result produced. Where a slight alteration turns that which was practically useless into what is useful and important, it is fit subject matter for a patent.(Lallubhai Chakkubhai v. Shamaldas Sankalchand Shah, AIR 1934 Bom 407).
08.03.06.07 A method of agriculture or horticulture is not patentable. Examples of subject matters excluded from patentability under this provision are: (a) A method of producing a new form of a known plant, even if it involved a modification of the conditions under which natural phenomena would pursue their inevitable course. (b) A method of producing improved soil from the soil with nematodes by treating the soil with a preparation containing specified phosphorathioates. (c) A method of producing mushrooms. (d) A method for cultivation of algae. Section 3(h)
08.03.06.08 Any process for the medicinal, surgical, curative, prophylactic, diagnostic, therapeutic or other treatment of human beings or any process for a similar treatment of animals to render them free of disease or to increase their economic value or that of their products is not patentable. This provision excludes from patentability, the following: (a) Medicinal methods: As for example a process of administering medicines orally, or through injectables, or topically or through a dermal patch. Section 3(i)
(b)
Surgical methods: As for example a stitch-free incision for cataract removal.
(c)
Curative methods: As for example a method of vaccination.
(d)
Prophylactic methods: As for example a method of cleaning plaque or stains from teeth.
(e)
Diagnostic methods: Diagnosis is the identification of the nature of a medical illness, usually by investigating its history and symptoms and by applying tests. Determination of the general physical state of an individual (e.g. a fitness test) is not considered to be diagnostic if it is not intended to identify or uncover pathology. Methods of diagnosis practised on the human or animal body is not patentable. For a claim to fall under this prohibition, it must include both the deductive step of making the diagnosis and preceding steps constructive for making that diagnosis involving specific interactions of a technical nature with the human or animal body. Methods of diagnosis performed on tissues or fluids which have been permanently removed from the body may not beexcluded.
(f)
Therapeutic methods: The term “therapy’’ includes prevention as well as treatment or cure of disease. Therefore, the process relating to therapy may be considered as a method of treatment and as such not patentable.
(g)
Any method of treatment of animal to render them free of disease or to increase their economic value or that of their products. As for example, a method of treating sheep for increasing wool yield or a method of artificially inducing the body mass of poultry.
(h)
Further examples of subject matters excluded under this provision are: any operation on the body, which required
the skill and knowledge of a surgeon and includes treatments such as cosmetic treatment, the termination of pregnancy, castration, sterilization, artificial insemination, embryo transplants, treatments for experimental and research purposes and the removal of organs, skin or bone marrow from a living donor, any therapy or diagnosis practiced on the human or animal body and further includes methods of abortion, induction of labour, control of estrus or menstrual regulation. (i) Application of substances to the body for purely cosmetic purposes is not therapy. (j) Patent may however be obtained for surgical, therapeutic or diagnostic instrument or apparatus. Also the manufacture of prostheses or artificial limbs and taking measurements thereof on the human body are patentable.
08.03.06.09 Plants and animals in whole or any part thereof other than micro-organisms but including seeds, varieties and species and essentially biological processes for production or propagation of plants and animals are not patentable. The subject matters excluded under this provision are: (a) plants in whole or in part (b) animals in whole or in part (c) seeds (d) varieties and species of plants and animals (e) essential biological process(es) for production or propagation of plants and animals. Microorganisms and microbiological processes are patentable. However a microorganism discovered from the nature is not patentable. A new process of preparation of a vaccine under specific Section 3(j)
scientific conditions, the vaccine useful for protecting poultry against contagious bursitis infection was held to be patentable by the Court on the ratio that the statute does not make a manner of manufacture as un-patentable even if the end products contains a living organism. (Dimminaco – A.G. Vs. Controller of Patents & Designs and Others). Plant varieties are provided protection in India under the provisions of the Protection of Plant Varieties and Farmers’ Rights Act, 2002.
08.03.06.10 A mathematical or business method or a computer programme per se or algorithms are not patentable. a. Under this provision, mathematical methods, business methods, computer programmes per se and algorithms are not considered as patentable inventions. In relation to computer programs, the law provides a qualification that what is not patentable is only computer program per se. b. ‘Mathematical methods’ are considered to be acts of mental faculty. A method of calculation, formulation of equations, finding square roots, cube roots and all other methods directly or indirectly involving mathematical methods are therefore not patentable. With the development in computer technology, these mathematical methods are used for writing algorithms and computer programs for different applications and the claimed invention is sometimes camouflaged as one relating to the technological development rather than the mathematical method itself. These methods, claimed in any form, if in substance relate to mathematical method are considered to be not Section 3(k)

patentable.

c.
“Business Methods” claimed in any form are not patentable subject matter. The term ‘Business Methods’ involves whole gamut of activities in a commercial or industrial enterprise relating to transaction of goods or services. With the development of Internet Technologies, business activities have grown tremendously through e-commerce and related B2B and B2C business. The claims are at times drafted not directly as business method but apparently with hitherto available technical features such as internet, networks, satellites, tele-communications, etc. The exclusions are carved out for all business methods and, therefore, if in substance the claims relate to business methods, even with the help of technology, they are not considered to be patentable.
d.
Claims directed at ‘computer programme products’ are computer programme per se stored in a computer readable medium and as such are not allowable.
e.
If a claim in a patent application is not directed at a computer programme per se it could be patentable, if all other patentability conditions are met. This provision thus necessitates distinguishing computer programmes per se from other types of inventions that uses or implements computer programmes.
f.
The computer programmes are often claimed in the form of algorithms as method claims or system claims with some ‘means’ indicating the function of flow charts or process steps. The algorithm related claims may be even wider than the computer programme claimed by itself, for a programme represents a particular set, the algorithm expresses the principles
generally and gives way for different programmes to be written based on the same algorithm and as such are not patentable. g. Essentially, all computer programmes need a combination with some hardware for their functionality. In an application for patent for a new hardware system, the possibility of a computer programme forming part of the claims cannot be ruled out. It has to be carefully considered as to how integrated is the novel hardware with the computer programme. Further, it is also to be considered whether the machine is programme specific or the programme is machine specific. A computer programme which may work on any general purpose known computer does not meet the requirement of patentability. h. Method claims, whether independent or dependent, reciting computer programs without process limitations in the form of hardware features are not allowable. For a method reciting computer programme to be patentable, it must clearly recite into it limiting hardware integers that enable the program to function. i. Claims directed at computer programs coupled to hardware, enabling the hardware to perform a certain function may be allowable, if such an invention meets all other conditions of patentability.
08.03.06.11 A literary, dramatic, musical or artistic work or any other aesthetic creation whatsoever including cinematographic works and television productions is not patentable. Writings, music, works of fine arts, paintings, sculptures, computer programmes, electronic databases, books, pamphlets, lectures, addresses, sermons, dramatic- Section 3(l)
musical works, choreographic works, cinematographic works, drawings, architecture, engravings, lithography, photographic works, applied art, illustrations, maps, plans, sketches, three-dimensional works relating to geography, topography, translations, adaptations, arrangements of music, multimedia productions, etc. are not patentable. Such works fall within the domain of the Copyright Act, 1957.
08.03.06.12 A mere scheme or rule or method of performing mental act or method of playing game is not patentable. A mere scheme or rule or method of performing mental act or method of playing game are excluded from patentability, because they are considered as outcome of mere mental process. For example, a. Method of playing chess. b. Method of teaching. c. Method of learning. d. Method of operating a machine or equipment as per a set of instructions. Section 3(m)
08.03.06.13 A presentation of information is not patentable. Any manner, means or method of expressing information whether visual, audible or tangible by words, codes, signals, symbols, diagrams or any other mode of representation is not patentable. For example, a speech instruction means in the form of printed text where horizontal underlining indicated stress and vertical separating lines divided the works into rhythmic groups is not patentable. For instance, railway time table, 100years calendar etc. In the matter of application No. 94/Cal/2002, the Controller held that the Patent system was meant for protecting only one kind of creativity i.e. technological creativity and since the claimed invention relates to business Section 3(n)
method and method of presenting information, it is not allowed.
08.03.06.14 Topography of integrated circuits is not patentable. Since protection of Layout Designs of Integrated Circuits is governed separately under the Semiconductor Integrated Circuit Lay-out Designs Act, 2000, three-dimensional configuration of the electronic circuits used in microchips and semiconductor chips is not patentable. Section 3(o)
08.03.06.15 An invention which in effect, is traditional knowledge or which is an aggregation or duplication of known properties of traditionally known component or components is not patentable. Traditional Knowledge, being knowledge already existing, is not patentable. An example is the anti-septic properties of turmeric for wound healing. Another example is the pesticidal and insecticidal properties of neem. The Examiner conducts an investigation by using the Traditional Knowledge Digital Library (TKDL) and other resources to decide as to whether the claimed subject matter falls within the purview of this provision. Section 3(p)
08.03.07 Sufficiency of Disclosure Sufficiency of disclosure is yet another aspect, which is checked by the Examiner while examining a patent application. The Examiner will look for whether: a. the specification is properly titled. b. the subject matter is fully and particularly described in the specification. c. the claims define the scope of the invention properly. d. the Specification describes the best method of performing the invention or not. e. the source and geographical origin, in case of Section 10
inventions related to biological materials, has been disclosed. f. approval obtained from Biodiversity Authority, wherever applicable. accession number and other details of the depository given, wherever applicable
08.03.08 Unity of Invention a) The Claims of a Specification shall relate to a single inventive concept. In case, an application comprises a plurality of inventive concepts the examiner refers to the same in his report. The application may be divided in order to meet the objection of plurality of distinct inventions. b) The determination whether a group of inventions is so linked as to form a single general inventive concept shall be made without regard to whether the inventions are claimed in separate claims or as alternatives within a single claim. c) Unity of invention between process and apparatus or means requires that the apparatus or means have been specifically designed for carrying out the process, or at least a step of the process. d) Independent claims of different categories may relate to a single inventive concept. For example: 1. Claims for a product and process specially adapted for manufacture of the product. 2. Claims for a process and apparatus or means specifically designed for carrying out the process. 3. A mould for casting an article, a method of making that mould, a process of casting the article by using the said mould will constitute Section 10(5)
a single invention. 4. A locking system containing plug and socket wherein separate independent claims for a plug and socket may constitute a single inventive concept. 5. A broadcasting system comprising transmitter and receiver. 6. If invention relates to a new type of spray bottle, claims may be directed to the spray bottle itself (a product) and a method of making the spray bottle (a process). 7. In case of a genetically modified Gene Sequence/ Amino Acid Sequence claims may be directed to a Gene sequence / Amino Acid sequence, a method of expressing the sequence, an antibody against that protein / sequence, a kit containing such antibody / sequence. 8. In case of a drug or pharmaceutical product, claims may be directed to a drug or pharmaceutical product, a process of making the product, a composition containing the drug.
08.04 Consideration of the Report by Controller and issuance of FER 1. The Controller considers the report of the examiner ordinarily within one month from the date of the receipt of such report and a gist of objections, if any, is sent to the applicant in the form of a report First Examination Report (FER) along with the application and specification, if required. If there is no objection to the grant of patent and no pre-grant Section 14, 15, 21
under Section 25(1) is pending, the patent is granted at the earliest. 2. The FER is sent to the applicant, even when the request for examination has been filed by the third party. An intimation is given to the third party. 3. First Examination Report may contain office objections relating to: a. Lack of novelty, inventive step and industrial applicability. b. Subject matter relating to a category, which falls within the purview of Section 3 and 4. c. Non-fulfillment of any other requirement under the Act. 4. The applicant is required to comply with all the requirements imposed upon him by the Act as communicated through FER or subsequent communication, at the earliest. However, if such requirements are not met within twelve months from the date of forwarding of FER to the applicant, the application is deemed to have been abandoned under Section 21(1). If for reasons beyond the control of the applicant, the applicant re-files the documents after twelve months from the date of issuance of FER, the Controller may decide to proceed with the case based on the evidence made available to him for such delay. 5. Further, no appeal lies against an order of abandonment under this Section. 6. When the applicant re-files the documents within twelve months the application has to be examined in a fresh manner by the examiner. Upon examination, if it is found that the requirements of the Act have
been met, the Patent is granted. 7. If the applicant contests any of the objections communicated to him by the Controller or he re-files his specification or other documents, along with his observations as to whether or not the specification is to be amended, an opportunity of being heard is given, if requested by the applicant. 8. After hearing the applicant, the Controller may specify or permit such amendment as he thinks fit and grant the patent. The Controller may refuse to grant the patent unless the amendments so specified are made or any other requirements of the Act and Rules are not complied with. 9. No refusal of patent is done without giving an opportunity of being heard under Section 15. An order refusing an application under Section 15 shall be a speaking order. Such an order under Section 15 is appeallable before the Intellectual Property Appellate Board.
08.05 Post-dating of Application Section 17
1. The application for patent may be post-dated to a date not later than six months from the date of application on a request made by the applicant at any time before the grant of patent along with the prescribed fee. i. Where such request is made before the examination of application, the same may be allowed and the examination shall be conducted with reference to the date so post-dated. ii. Where such request is made after the issuance of First Examination Report, the same may be allowed subject to fresh examination with reference to the date so post-dated.
2. If the application or specification (or drawings if any) or any document is required to be amended under section 15 to comply with the requirements of the Act or the Rules, the Controller may direct that the application or specification or other documents related thereto be deemed to have been made on the date on which the requirements are complied with or the date on which it is re-filed after complying with the requirements. In case this provision is invoked, the application shall be examined afresh with reference to the date as directed by the Controller. 3. Following may be instances in which Section 17(2) may be invoked by the Controller: i. A missing part is brought-in through an amendment, for the purpose of meeting the office objections during Section 15 proceedings, by the applicant. ii. A new drawing is brought-in through an amendment for the purpose of explaining the amended specification, and without such amended drawing the amended specification cannot be explained.
08.06 Pre-Grant Opposition Section 25(1) Rule 55
a. Any person can file opposition by way of representation to the Controller against the grant of Patent, at the appropriate office, any time after publication of patent application u/s 11A, but before the grant of Patent on any of the grounds mentioned in Section 25(1). The date of grant of Patent is the date on which the Controller orders the grant of patent in the file. Simultaneously, the
patent number is generated and the fact of grating the patent is available on the official website.
b. A Patent is not granted before the expiry of six months from the date of publication under S.11A. Therefore, a person should file the pre-grant opposition within the assured period of six months from the date of Publication, to make sure that the pre-grant opposition is filed before the grant of patent.
c. The representation shall include a statement and evidence, if any, in support of such representation and a request for hearing, if so desired.
d. The Controller shall consider the representation only after a Request for Examination for that Application has been filed.
e. The Pre-Grant Opposition, if available on record, is considered by the Controller along with the report of the Examiner.
f. On consideration of the opposition, if the Controller is of the opinion that the opposition is devoid of any merit, an opportunity of hearing shall be granted to the opponent, if requested. After hearing the opponent, if the Controller is still of the opinion that the opposition shall be refused, a speaking order shall be issued rejecting the pre-grant opposition, ordinarily within one month.
g. However, if the Controller is of the opinion that pre-grant opposition has merit and the application shall be refused or amended, a notice is given to the applicant along with a copy of the representation. The applicant shall, if he so desires, give reply to that representation along with his statement and evidence, if any, in support of his application within three months form the date of the notice. h. The Controller shall consider the statement and evidence filed by the applicant and may either refuse the grant of patent or ask for amendment of the complete specification to his satisfaction before the grant of patent. i. After considering the representation and submissions made during the hearing, the Controller shall proceed further simultaneously, either rejecting the representation and granting the patent or accepting the representation and refusing the grant, ordinarily within one month from the completion of the above proceedings. If the application for patent is to be refused on consideration of the pre-grant opposition u/s 25(1), a speaking order of refusal shall be issued under Section 15.
08.07 Grant Section 43
08.07.01 Compliance of conditions under the Act The Patent is granted as expeditiously as possible when 1. the application has not been refused by the Controller
by virtue of any power vested in him by this Act, or 2. the application has not been found to be in contravention of any of the provisions of the Act, or For instance, a. All objections raised by the examiner have been met and documents returned with the FER have been resubmitted after complying with the requirements, within 12 months from the date of the FER. b. When there is no pre-grant representation pending before the grant of Patent or when the Pre-Grant Opposition has been disposed of in favor of the applicant. It may be noted that the date of grant of patent is the date on which the patent number is generated and the patent is granted by the Controller.
08.07.02 Consequences of grant a. On the grant of patent, every patent is given a serial number. A Certificate of Patent is generated in the prescribed format and an entry in the e-register is made simultaneously. b. The complete specification as granted is made available to public through official website. c. The application, specification and other related documents are open for public inspection on payment of prescribed fee. d. The fact that the patent has been granted is published in the official journal of the Patent Office. e. A post-grant opposition under section 25(2) can be filed by any person interested within 12 months from the date of publication of grant. f. Every patentee and licensee has to furnish a statement regarding the working of the patented invention on commercial scale in India at regular Form-27
intervals (not less than six months) in the prescribed format.
08.07.03 Date of Patent a. The date of Patent is the date of filing of the Application. However, in case of a PCT National Phase application, the date of filing is the international filing date. b. The time for payment of renewal fee is reckoned from the date of recordal in the register of patents. However, since the introduction of electronic processing of patent applications, the date of recordal is same as the date of generation of Certificate of Patents. Section 43
08.07.04 Conditions subject to which a Patent is granted a. any machine, apparatus or other article in respect of which the patent is granted or any article made by using a process in respect of which the patent is granted, may be imported or made by or on behalf of the government for the purpose merely of its own use; b. any process in respect of which the patent is granted may be used by or on behalf of the government for the purpose merely of its own use; c. any machine, apparatus or other article in respect of which the patent is granted or any article made by the use of the process in respect of which the patent is granted, may be made or used, and any process in respect of which the patent is granted may be used, by any person, for the purpose merely of experiment or research including the imparting of instructions to pupils; and d. in the case of a patent in respect of any medicine or Section 47
drug, the medicine or drug may be imported by the government for the purpose merely of its own use or for distribution in any dispensary, hospital or other medical institution maintained by or on behalf of the government or any other dispensary, hospital or medical institution which the Central Government may, having regard to the public service that such dispensary, hospital or medical institution renders, specify in this behalf by notification in the Official Gazette.
08.07.05 Rights of Patentee Section 48
a. In case of a patented product, the patentee shall have the exclusive right to prevent third parties, from the act of making, using, offering for sale, selling or importing for those purposes that product in India. b. In case of a patented process, the patentee shall have the exclusive right to prevent third parties, from the act of using that process, and from the act of using, offering for sale, selling or importing for those purposes the product obtained directly by that process in India.
08.07.06 Rights of co-owners Section 50
a. Where a patent is granted to two or more persons, each of those persons shall, unless an agreement to the contrary is in force, be entitled to an equal undivided share in the patent. b. Subject to the provisions contained in this section and in Section 51, where two or more persons are registered as grantee or proprietor of a patent, then, unless an agreement to the contrary is in force, each of those persons shall be entitled, by himself or his agents, to the rights conferred by Section 48 for his
own benefit without accounting to the other person or persons. c. Subject to the provisions contained in this section and in section 51 and to any agreement for the time being in force, where two or more persons are registered as grantee or proprietor of a patent, then, a licence under the patent shall not be granted and a share in the patent shall not be assigned by one of such persons except with the consent of the other person or persons. d. Where a patented article is sold by one of two or more persons registered as grantee or proprietor of a patent, the purchaser and any person claiming through him shall be entitled to deal with the article in the same manner as if the article had been sold by a sole patentee. e. Subject to the provisions contained in this section, the rules of law applicable to the ownership and devolution of movable property generally shall apply in relation to patents, and nothing contained in subsection (1) or sub-section (2) shall affect the mutual rights or obligations of trustees or of the legal representatives of a deceased person or their rights or obligations as such. f. Nothing in this section shall affect the rights of the assignees of a partial interest in a patent created before the commencement of this Act.
08.07.07 Patent rights not infringed in certain cases Section 49
Where a vessel or aircraft registered in a foreign country or a land vehicle owned by a person ordinarily resident in such country comes into India (including the territorial waters thereof) temporarily or accidentally only, the rights
conferred by a patent for an invention shall not be deemed to be infringed by the use of the invention— i) in the body of the vessel or in the machinery, tackle, apparatus or other – accessories thereof, so far as the invention is used on board the vessel and for its actual needs only; or ii) in the construction or working of the aircraft or land vehicle or of the accessories thereof, as the case may be. These provisions shall not extend to vessels, aircraft or land vehicles owned by persons ordinarily resident in a foreign country the law of which do not confer corresponding rights with respect to the use of inventions in vessels, aircraft or land vehicles owned by persons ordinarily resident in India while in the ports or within the territorial waters of that foreign country or otherwise within the jurisdiction of its courts.
08.07.08 Term of Patent Section 53
The term of Patent is 20 years from the date of filing of application in respect of all the patents, including those for which the term had not expired on 20th May, 2003, when Patent (Amendment) Act 2002 came into force, provided that the renewal fee is paid every year before the due date or within the extended period (maximum six months).

Chapter-9 Post-grant opposition

09.01 Post-grant opposition Section 25(2) Rule 55A
1. Any interested person can file a Notice of Opposition against the grant of Patent in the prescribed format, in duplicate, within twelve months from the date of publication of grant of patent at the appropriate Office. 2. The opponent must prove as to how he is an interested person. 3. Interested person includes a person engaged in, or in promoting research in the same field as that to which the invention relates. It may be an organization that has a manufacturing or trading interest in the goods connected with the patented article or which has a financial interest in manufacturing such goods or which possesses Patents relating to the same subject. 4. The post-grant opposition can be filed on the grounds as mentioned in Section 25(2), but no other grounds. 5. After receipt of Notice of Opposition, the Controller shall notify the patentee about the fact of receipt of such notice, without any delay. 6. Copies of all documents which are referred to in the Notice of Opposition or in any statement or evidence filed in connection with the opposition should be furnished in duplicate and duly authenticated. A Form 7 Section 2(1)(t)
copy of the statement and evidence, if any, shall be
delivered to the patentee by the opponent.
7. Such documents should be filed either in original or
should be duly notarized.
8. If the patentee desires to contest the opposition, he
shall file a reply statement setting out fully the
grounds upon which the opposition is contested, and
evidence if any, in support of his case within a
period of two months from the date of receipt of the
copy of opponent’s written statement and evidence,
if any, and deliver a copy to the opponent.
9. If the patentee does not desire to contest or does not
file his reply and evidence within two months, the
patent shall be deemed to have been revoked and the
Controller shall issue the order of revocation of
Patent and the fact of revocation is entered in the
register of patents.
10. After receipt of reply from the patentee, the
opponent may file his reply statement and evidence
within one month from the date of delivery to him
of a copy of the patentee’s reply statement and
evidence. Reply evidence of the opponent shall be
strictly confined to the matters in the patentee’s
evidence. The opponent shall deliver a copy of his
reply statement and evidence to the patentee.
11. No further evidence shall be delivered by either
party, except with the leave or direction of
Controller.
12. With respect to further evidence filing, either party Rule 60
shall do so before the Controller’s notification on
the fixation of the date of hearing.
13. Where a specification or other document in a
language other than English is referred to in the notice, statement or evidence, an attested translation thereof in duplicate in English should be furnished along with such notice, statement or evidence, as the case may be.
09.02 Constitution of Opposition Board Rule 56-60
1. After receipt of Notice of Opposition, an Opposition Board is constituted by the Controller, by order, to examine such notice including all the documents filed under rule 57-60 in connection with opposition by the opponent as well as patentee. 2. The Board shall submit the report with reasons on each ground taken in the Notice of Opposition after examining all statements, documents and evidence submitted by the parties, as a joint recommendation within three months from the date on which all such documents were forwarded to them. 3. The Opposition Board consists of three members with one of them as Chairman. 4. The examiner may be a member of the Board. But, the examiner who has dealt with the application for patent during the prosecution proceedings for grant of patent thereon shall not be included as a member of the Board. 5. If any further evidence is taken on record by the Controller, by an order in writing, the same shall be forwarded to the Opposition Board for their consideration. This shall also apply when such further evidence is taken on record after the receipt of report from Opposition Board.
09.03 Hearing Rule 62
1. After receiving the recommendation of Opposition Board the Controller shall fix without undue delay, a date and time for the hearing of the opposition and inform the parties, at least ten days in advance. 2. On receipt of the notice of hearing, if either party desires to be heard, he shall inform the Controller by a notice along with the prescribed fee. 3. The Controller may require the members of Opposition Board to be present in the hearing 4. The Controller may refuse to hear any party which has not given such notice and fee. 5. If either party intends to rely on any Publication at the hearing not already mentioned in the notice of opposition, statement or evidence, he shall give to the other party and to the Controller a notice of his intention to do so, together with details of such publication. Such notice shall be given at least five days before the date of hearing. 6. After hearing the party or parties desirous of being heard, or if neither party desires to be heard, then without a hearing, and after taking into consideration the recommendation of Opposition Board, the Controller shall decide the opposition, i.e., he may revoke the patent, or order amendments in the Patent or refuse the opposition and issue a speaking order. 7. If amendment of specification or any other document is ordered by the Controller, the patentee shall submit such amended documents to the office within a reasonable time, as directed by the Controller.

Chapter - 10 Post-grant procedures

10.01 Maintenance of Patent – renewal
To keep a patent in force, the renewal fees shall be payable Section 53,
at the expiration of the second year from the date of the 142 (4)
patent or of any succeeding year and the same shall be remitted to the patent office before the expiration of the second or the succeeding year. Rule 80
Further, renewal of a patent can be done beyond the due date in the extended period of six months from that date by filing Form-4 along with the prescribed fee. This period of six months is not extendable. a. Where a patent is granted later than two years from the date of filing of the application, the fees which have become due in the meantime may be paid within a period of three months from the date of the recording of the patent in the register. The date of recording of patent in the register of patents is communicated to the applicant through a letter which accompanies the Certificate of Patent. The period for payment of renewal fees may be extended by a maximum of six months by filing Form-4 with the required fee. b. If a patent is granted before the expiry of two years from the date of filing of application, the first renewal fee becomes due in respect of the third year. However, the renewal fee shall be paid before the expiry of second year. This period is extendable by maximum six months. c. First Renewal fee for a patent becomes due in respect of the third year counted from the date of filing of Form-4
application for patent. However, the renewal fee for third year has to be paid before the expiry of second year. For clarification, the renewal fee may be paid before the expiration of the fourth year from the date of Patent for the fifth year and so on. d. When a patent is granted after the expiry of two years from the date of filing, the renewal fee that has become due with respect to the third and subsequent years shall be paid within a period three months from the date of recording of patent in the register. e. While paying the renewal fee, the patentee shall quote correctly the patent number, date of patent and the year in respect of which the renewal fee is being paid. Any mistake in the above said particulars may lead to a lapse of patent. f. For the Patent of Addition no renewal fee is required to be paid.
10.02 Restoration of Lapsed Patents
a. When a Patent has ceased to have effect due to non- Section 60
payment of renewal fees within the prescribed time, the Rule 84
Patent may be restored by filing an application for restoration, in Form 15, within eighteen months from the date on which the patent ceased to have effect. Such an application can be made by the patentee / assignee, or his legal representative and in case of joint applicants, then, with the leave of the Controller, any one or more of them without joining the others. b. The applicant has to state, the circumstances which led to the failure of payment of renewal fees. The application must include a statement fully setting out the circumstances that led to the failure to pay the renewal fee. This statement may be supported by Form-15
evidence and copies of any documents referred to. c. The evidence must support the patentee's claim that the failure to pay was unintentional and there has been no undue delay in applying for restoration. d. The Controller may call for further evidence to justify that the failure to pay was unintentional and that there has been no undue delay for making the application. e. If a patentee has failed to register a change of name before cessation he must first apply under Rule 94 for alteration in the register. If he changed his name after cessation he must prove his identity. In both cases he must draw and sign the application in his new name but in the latter case must add 'formerly known as' to his identification. Rule 94
10.02.01 Procedure for disposal of application for restoration Section 60-62
a. When the Controller is prima facie satisfied that the failure to pay renewal fee was unintentional and there Rule 84-86
had been no undue delay, the application for restoration will be published in the official journal. b. If the Controller is satisfied that a prima facie case for restoration has not been made, the Controller may issue a notice to the applicant to that effect. Within one month from the date of notice, if the applicant makes a request to be heard on the matter, a hearing shall be given and the restoration application may be disposed. If no request for hearing is received within one month from the date of notice by the Controller, the application for restoration is refused. In case of rejection of the application for restoration, a speaking order shall be issued. c. Any person interested may give Notice of Opposition, in the prescribed manner, to the application within two Form-14
d. e. f. g. h. months of the date of Publication in the official journal on the grounds that the failure to pay the renewal fee was not unintentional or that there has been undue delay in the making of the application. The Notice of Opposition shall include a statement setting out the nature of the opponent's interest, the grounds of opposition, and the facts relied upon. The notice of opposition shall be sent to the applicant expeditiously. The procedure specified in rules 57 to 63 for post grant opposition for filing of written statement, reply statement, reply evidence, hearing and cost shall apply in this case. [See Chapter 9 - Post Grant Opposition] When no opposition is received within a period of two months from the date of publication, or opposition, if any, is disposed off, the Controller shall issue an order allowing the application for restoration. The unpaid renewal fee and the additional fee, as mentioned in the first schedule, shall be paid within one month from the date of order of the Controller. The fact that a patent has been restored shall be published in the official journal. To protect the persons who have begun to use the applicant's invention between the date when the Patent ceased to have effect and the date of Publication of the Application for restoration, every order for restoration includes the provisions and other conditions, as the Controller may impose, for protection and compensation of the above-mentioned persons. No suit or other proceeding shall be commenced or prosecuted in respect of an infringement of a Patent committed between the date on which the Patent ceased to have First schedule
effect and the date of the Publication of the Application for restoration of the patent.
10.03 Registration of assignments/Transfer of Right
a. An assignment of a patent or of a share in a patent, a Section 68, 69
mortage, licence or the creation of any other interest in a patent shall be valid only if the same were in writing and the agreement between the parties concerned is reduced to the form of a document embodying all the terms and conditions governing their rights and obligations and has been duly executed. b. Any person who becomes entitled by assignment, Rule 90-92
transmission or operation of law to a patent or to a share in patent or becomes entitled as a mortgagee, licencee or otherwise to any other interest in a patent, may apply in writing in Form-16 to the Controller, for the registration of his title or as the case may be of notice of his interest in the register. Such an application can also be made by the assignor, mortgagor, licensor or other party as the case may be. c. Where such application is made for the registration of title or notice of interest of any person, the Controller, upon proof of title or interest to his satisfaction, shall enter in the register such particulars as are appropriate. d. If there is any dispute between the parties the Controller may refuse to take any action to make an entry in the register, until the rights of the parties have been determined by a competent court. e. Except for the purpose of making an application for registration of right, title or interest in the register of patents or for an application for rectification of the register of patents in the IPAB, a document in respect of which no entry has been made in the register shall Form-16
not be admitted by the Controller or by any court as evidence of the title of any person to a patent or to a share or interest therein unless the Controller or the court, for reasons to be recorded in writing, otherwise directs.
10.04 Surrender Of Patents
a. The patentee may at any time offer to surrender his patent through an application on plain paper. On receipt of such an offer the Controller publishes the offer in the Official Journal and also notifies every person (other than the patentee) whose name appears in the register as having an interest in the patent. b. An opposition against the offer to surrender the patent may be filed by any person interested in Form 14 (in duplicate) within 3 months from the date of publication of such notice in the Official Journal. The Controller shall inform the Patentee on receipt of such notice. c. The procedure relating to filing of written statement, reply statement leaving evidence and hearing of the opposition is similar to that of the opposition to the grant of Patents [See Chapter 9 -Post-Grant Opposition]. d. In case, the Controller accepts the patentee's offer to surrender the patent, he may direct the patentee to return the patent and on the receipt of which, the Controller shall by order, revoke it and publish such revocation in the official journal. Section 63 Rule 57- 63
10.05 Working of Patents Section 83
a. Patents are granted for the purpose of encouraging inventions, which will enhance industrial development
and, therefore, should be worked in its fullest extent within the territory of India. Patents are granted to encourage inventions and to secure that the inventions are worked in India on a commercial scale and to the fullest extent reasonably practicable without undue delay.
b. The Controller has the power to call for the information Section 146(2)
such as periodical statements as to the extent to which the patented invention has been commercially worked in India, as may be specified in the notice issued to that effect at any time during the continuance of the Patent c. A patentee or a licensee shall furnish such information within two months from the date of such notice or within such further time as the Controller may allow. d. The patentee and every licensee shall furnish the details of working of the invention in Form 27 in respect of every calendar year within three months of the end of each year. Rule 131(2)
10.06 Amendments after the grant of patents
a. After the grant of patent, the patentee may apply in Sections 57,
Form-13 for an amendment of the application for 59
patent, complete specification or any document relating Rule 81-83
thereto to be amended subject to such conditions, if any, as the Controller thinks fit. Such a request may be filed Form-13 with prescribed fee. Such request may also be made for amendment of priority date. b. The request shall state the nature of the proposed amendment, highlighted in an annexed copy along with the reasons. The amendments are allowable only by Form-13
c. d. e. f. way of disclaimer, correction or explanation. Such amendments shall be only for the purpose of incorporation of actual fact. Further, no amendment of a complete specification shall be allowed the effect of which would be that the specification as amended would claim or describe matter not in substance disclosed or shown in the specification before the amendment, or the amended claim(s) do not fall wholly within the scope of claim(s) of the specification before the amendment. An application for amendment may be published along with the nature of proposed amendment. However, if the nature of proposed amendment is substantive, the application for amendment shall be published. For instance, any application for amending the complete specification or the claims or the application for patent shall be published. The amended pages have to be filed in duplicate by the applicant along with duly cancelled original pages. Any person interested may file a notice of opposition in Form-14 within three months from the date of publication of the application for amendment. Where such a notice of opposition is filed, the Controller notifies the applicant for amendment. After giving an opportunity to the applicant and opponent, if any, the Controller shall dispose off the case. The procedure specified in rules 57 to 63 for post grant opposition for filing of written statement, reply statement, reply evidence, hearing and costs shall apply Section 64(1)(o) Section 64(1)(0) Rules 57 to 63
in this case. [See Chapter 9 - Post Grant Opposition] g. Amendments allowed after the grant of patent shall be published. h. A leave to amend the complete specification obtained by fraud is a ground for revocation of patent under Section 64. i. If any suit for infringement is pending before a Court or any proceeding for revocation of the Patent is pending before the High Court, the Controller shall not pass any order allowing or refusing the application for amendment.
10.07 Procedure to be followed, after the grant of patent, in case of death of an applicant Section 44
a. If the applicant had died before the grant of patent, but the patent was granted in his name, a person in whose name the patent ought to have been granted may make a request to the Controller. The Controller may amend the patent by substituting the name of the deceased applicant with the name of such claimant. Such a request has to be made in Form-10. b. If the applicant dies after the patent has been granted, any person who becomes entitled to the patent or to a share in the patent, by operation of law, may make a request for registration of his title. Such a request shall be made in Form-16.
10.08 Register of Patents Section 67
a. A register of patents is available to public at each Patent Office location and contains details about the patentee, Section 72
notifications in respect of assignments, transmissions, patents, licences under patents, and amendments,
b. extension and revocations of patents. A copy of, or extracts from, the register of patents,
certified to be a true copy under the hand of the Controller is admissible in evidence in all legal
c. proceedings. Upon grant of a patent, the Controller enters in the Rule 94
Register of Patents the names, addresses and nationality of the patentee, title of the invention including the
categories to which the invention relates, date of the patent, date of granting thereof and address for service
of the patentee. The fact of payment of renewal fee shall also be entered in the register of patents.
d. The Controller shall also enter in the register of patents, particulars regarding proceedings under the Act, before
the Controller or in the Courts in respect of every
e. patent. An application for alteration of name, nationality,
address or address for service as entered in the register of patents may be made to the Controller in respect of
any Patent. The Controller may require such proof of the alteration as he may think fit before acting on the request. If the Controller allows such a request, entries
f. in the register are altered accordingly. If a patentee makes a request in writing along with the prescribed fees for entering an additional address for
service in India and the Controller is satisfied that the
request shall be allowed, the additional address shall be entered in the register. However, a maximum of two
addresses for service shall be entered in the register at any one time.
g. The register of patents is available to the public on the official website.
10.09 Rectification of register of patents Section 71
a. An application for rectification of register of patents may be made to the Intellectual Property Appellate Board (IPAB) by any person aggrieved: i. by the absence or omission from the register of any entry; or ii. by any entry made in the register without sufficient cause; or iii. by any entry wrongly remaining on the register; or iv. by any error or defect in any entry in the register. b. Notice of such application made before the IPAB is given to the Controller, who is entitled to be heard on the application. Further, if so ordered, the Controller shall appear before the IPAB. c. If IPAB passes any order rectifying the register, a notice of rectification is served upon the Controller, who, upon such receipt, rectifies the register.

Chapter-11 Appeals

11.01 Appellate Board and Appeals
11.01.01 Appellate Board a. An Appellate board, established under the Section 83 of the Trade Marks Act, 1999, is the Appellate Board for deciding appeals to the decisions of the Controller. The Board has been constituted for speeding up legal proceedings. b. The Appellate Board became operational on and from 2nd April, 2007 vide S.O. 507(E) published in the Gazette of India, Part II, Section 3, Sub-section (ii) Section 116
11.01.02 Appeals a. An appeal lies from the decision, order or direction made or issued under the Patents Act by the Central Government or any act or order of the Controller for the purpose of giving effect to any such decision order or direction. b. Further, an appeal shall lie from any decision, order or direction of the Controller or Central Government under Sections 15, 16, 17, 18, 19, 20, 25(4), 28, 51, 54, 57, 60, 61, 63, 66, 69(3), 78, 84(1) to (5), 85, 88, 91, 92 and 94 of the Patents Act. c. No appeal lies from an Order of the Controller granting extension of time when such extension is provided in any provision of the Act or of the rules. Section 117A(2) Section 81
11.02 Appeal procedure Section 117[E & F]
a. Every appeal from the decision of the Controller under relevant sections as mentioned in Section 117A (2) shall be
accompanied by the certified copy of the decision, order or direction appealed against. Such application shall be in the
b. form and fee prescribed. Every appeal should be made within three months from the
date of the decision, order or direction of the Controller or
of the Central Government, as the case may be or within
such further time as the Appellate Board may in accordance with the Rules made by it allow.
c. The Controller shall have the right to appear and be heard- (a) in any legal proceedings before the Appellate Board in
which the relief sought includes alteration or rectification of the register or in which any question relating to the
practice of the patent office is raised; (b) in any appeal to the Appellate Board from an order of
the Controller on an application for grant to a patent— (i) which is not opposed, and the application is either refused by the Controller or is accepted by him
subject to any amendments, modifications, conditions or limitations, or
(ii) which has been opposed and the Controller considers that his appearance is necessary in the
d. public interest. The Controller shall appear in any case, if so directed by
e. the Appellate Board. The Controller may, in lieu of appearing, unless the
Appellate Board otherwise directs, submit a statement in writing signed by him, giving such particulars as he thinks
proper of the proceedings before him relating to the matter in issue or of the grounds of any decision given by him or
of the practice of the patent office in like cases, or of other

matters relevant to the issues and within his knowledge as the Controller may deem it necessary, and such statement shall be evidence in the proceedings.

Chapter-12 Revocation of Patent

12.01 Revocation of Patent
12.01.01 Revocation before High Court or Appellate Board a. Any person interested including the Central Government may make a petition on any of the grounds, specified for revocation of Patent under Section 64 of the Patents Act, before the Appellate Board. A Patent may also be revoked by the High Court on a counter-claim in a suit for infringement of patent. b. Grounds for revocation before the Appellate Board as well as the High Court are elaborated in Section 64. c. Without prejudice to the provisions contained in (a) above, a patent may be revoked by the High Court on the petition of the Central Government, if the High Court is satisfied that the patentee has without reasonable cause failed to comply with the request of the Central Government to make, use or exercise the patented invention for the purposes of government within the meaning of Section 99 upon reasonable terms. Section 64,
12.01.02 Revocation by Controller on direction of Central Government a. Where at any time after grant of a patent, the Central Government is satisfied that a patent is Section 65
for an invention relating to atomic energy for which no patent can be granted under sub-section (1) of section 20 of the Atomic Energy Act, 1962 (33 of 1962), it may direct the Controller to revoke the patent, and thereupon the Controller, after giving notice, to the patentee and every other person whose name has been entered in the register as having an interest in the patent, and after giving them an opportunity of being heard, may revoke the patent. b. In such proceedings, the Controller may allow the patentee to amend the complete specification in such manner as he considers necessary instead of revoking the patent.
12.01.03 Revocation by Central Government Where the Central Government is of opinion that a patent or the mode in which it is exercised is mischievous to the State or generally prejudicial to the public, it may, after giving the patentee an opportunity to be heard, make a declaration to that effect in the Official Gazette and thereupon the patent shall be deemed to be revoked. Section 66
12.01.04 Revocation by Controller for non-working Section 85
a. Where, in respect of a patent, a compulsory licence has been granted, the Central Government or any person interested may, apply to the Controller for an order revoking the patent on the ground: i. that the patented invention has not been worked in the territory of India, or ii. that reasonable requirements of the public
with respect to the patented invention have not been satisfied, or
iii. that the patented invention is not available to the public at a reasonably affordable
c. price. Such an application can be made only after the
expiration of two years from the date of the order granting the first compulsory licence.
d. Such an application shall contain such particulars as may be prescribed, the facts upon which the
application is based, and, in the case of an application other than by the Central
Government, shall also set out the nature of the
applicant's interest.
e. Such applications shall ordinarily be decided within one year of its being presented to the
Controller.

Chapter-13 Compulsory Licensing

13.01 Working of patents - General principles
a. Patents are granted to encourage inventions and to secure that the inventions are worked in India on a commercial scale and to the fullest extent that is reasonably practicable without undue delay. b. Patents are not granted merely to enable patentees to enjoy a monopoly for the importation of the patented article. c. The protection and enforcement of Patent rights contribute to the promotion of technological innovation and to the transfer and dissemination of technology, to the mutual advantage of producers and users of technological knowledge and in a manner conducive to social and economic welfare, and to a balance of rights and obligations. d. Patents granted do not impede protection of public health and nutrition and should act as instrument to promote public interest specially in sectors of vital importance for socio-economic and technological development of India. e. Patents granted do not in any way prohibit Central Government in taking measures to protect public health. f. The Patent right shall not be abused by the patentee or person deriving title or interest on Patent from the patentee, and the patentee or a person deriving title or interest on Patent from the patentee does not resort to practices which unreasonably restrain trade or adversely Section 83, 89
affect the international transfer of technology. g. Patents are granted to make the benefit of the patented invention available at reasonably affordable prices to the public. h. Patented inventions are worked on a commercial scale in the territory of India without undue delay and to the fullest extent that is reasonably practicable; i. The interests of any person for the time being working or developing an invention in the territory of India under the protection of a patent are not unfairly prejudiced.
13.02 Compulsory License Chapter XVI
An application for grant of a compulsory license may be made under the following provisions: a. Section 84. b. Section 91. c. Section 92. d. Section 92A.
13.02.01 Compulsory license under Section 84 Section 84
a. Any person interested may make an application to the Controller for grant of Compulsory Licence for a patent after the expiry of three years from the “date of grant” of the patent on the following grounds: i. that the reasonable requirements of public with respect to the patented invention have not been satisfied, or ii. that the patented invention is not available to the public at reasonably affordable price, or iii. that the patented invention is not worked in the territory of India. Such an application may also be made by the licensee. b. No person shall be estopped from alleging the grounds i-iii above by reason of any admission made by him in the Form-17
licence or otherwise or by reason of his having accepted such a licence. c. In considering such an application, the Controller shall take into account-i. the nature of the invention, the time which has elapsed since the sealing of the patent and the measures already taken by the patentee or any licensee to make full use of the invention; ii. the ability of the applicant to work the invention to the public advantage; iii. the capacity of the applicant to undertake the risk in providing capital and working the invention, if the application were granted; iv. as to whether the applicant has made efforts to obtain a licence from the patentee on reasonable terms and conditions and such efforts have not been successful within a reasonable period as the Controller may deem fit. Reasonable period shall be construed as a period not ordinarily exceeding a period of six months. However, these circumstances shall not be applicable in case of national emergency or other circumstances of extreme urgency or in case of public noncommercial use or on establishment of a ground of anti-competitive practices adopted by the patentee.
13.02.01.01 Reasonable requirements of the public Section 84(7)
The reasonable requirements of the public shall be deemed not to have been satisfied— I. if, by reason of the refusal of the patentee to grant a licence or licences on reasonable terms,— i. an existing trade or industry or the
development thereof or the establishment of
any new trade or industry in India or the
trade or industry in India or the trade or
industry of any person or class of persons
trading or manufacturing in India is
prejudiced; or
ii. the demand for the patented article has not
been met to an adequate extent or on
reasonable terms; or
iii. a market for export of the patented article
manufactured in India is not being supplied
or developed; or
iv. the establishment or development of
commercial activities in India is prejudiced;
or

II. if, by reason of conditions imposed by the patentee upon the grant of licences under the patent or upon the purchase, hire or use of the patented article or process, the manufacture, use or sale of materials not protected by the patent, or the establishment or development of any trade or industry in India, is prejudiced; or

III. if the patentee imposes a condition upon the grant of licences under the patent to provide exclusive grant back, prevention to challenges to the validity of patent or coercive package licensing; or

IV.
if the patented invention is not being worked in the territory of India on a commercial scale to an adequate extent or is not being so worked to the fullest extent that is reasonably practicable; or
V.
if the working of the patented invention in the territory of India on a commercial scale is being
prevented or hindered by the importation from abroad of the patented article by— i. the patentee or persons claiming under him; or ii. persons directly or indirectly purchasing from him; or iii. other persons against whom the patentee is not taking or has not taken proceedings for infringement.
13.02.01.02 Contents of application Such an application shall contain a statement setting out the nature of the applicant's interest, the facts upon which the application is based and the terms and conditions of the licence the applicant is willing to accept. Section 84 Form-17
13.02.01.03 Procedure Section 86-90
a. Where the Controller is satisfied, upon consideration of an application for compulsory licence, that a prima facie case has been made out, he shall direct the applicant to serve copies of the application upon the patentee and any other person appearing from the register to be interested in the patent in respect of which the application is made, and shall publish the application in the official journal. Rule 96-101
b. The patentee or any other person desiring to oppose the application may, within two months from the date of publication of the application or within such further time as the Controller may on application (made either before or after the expiration of the prescribed time) allow, give to the Controller notice of opposition. c. The notice of opposition shall include grounds on which the application is opposed and the terms and conditions of the licence, if any, the opponent is prepared to grant Form-14
to the applicant and shall be accompanied by evidence in support of the opposition. d. The opponent shall serve a copy of his notice of opposition and evidence on the applicant and notify the Controller when such service has been effected. e. No further statement or evidence shall be delivered by either party except with the leave of or on requisition by the Controller. f. The Controller shall forthwith fix a date and time for the hearing of the case and shall give the parties not less then ten days' notice of such hearing. g. The procedure specified in sub-rules (2) to (5) of rule 62, shall, so far as may be, apply to the procedure for hearing under this rule as they apply to the hearing in opposition proceedings. h. If, upon consideration of the evidence, the Controller is satisfied that a prima facie case has not been made out, he shall notify the applicant accordingly, and unless the applicant requests to be heard in the matter, the Controller shall refuse the application. The applicant shall make such a request within one month from the date of such notification. i. If the applicant requests for a hearing within the time allowed, the Controller shall, after giving the applicant an opportunity of being heard, determine whether the application may be proceeded with or whether it shall be refused and issue a speaking order on the matter as expeditiously as possible. Rule 62
13.02.01.04 Terms and conditions Section 90
In settling the terms and conditions of a licence, the Controller endeavours to secure -a. that the royalty and other remuneration, if any, reserved
to the patentee or other person beneficially entitled to the patent, is reasonable, having regard to the nature of the invention, the expenditure incurred by the patentee in making the invention or in developing it and obtaining a patent and keeping it in force and other relevant factors; b. that the patented invention is worked to the fullest extent by the person to whom the licence is granted and with reasonable profit to him; c. that the patented articles are made available to the public at reasonably affordable prices; d. that the licence granted is a non-exclusive licence; e. that the right of the licensee is non-assignable; f. that the licence is for the balance term of the patent unless a shorter term is consistent with public interest; g. that the licence is granted with a predominant purpose of supply in the Indian market and that the licensee may also export the patented product, if need be in accordance with the provisions of sub-clause (iii) of clause (a) of sub-section (7) of section 84; h. that in the case of semi-conductor technology, the licence granted is to work the invention for public noncommercial use; i. that in case the licence is granted to remedy a practice determined after judicial or administrative process to be anti-competitive, the licensee shall be permitted to export the patented product, if need be. The terms and conditions of a licence settled by the Controller, may be revised upon Application by the licensee after a period of not less than twelve months, on the ground that the terms and conditions settled have proved to be more onerous than originally expected and that in consequence
thereof the licensee is unable to work the invention except at a loss. However, no such application shall be entertained a second time.
13.02.01.05 Application by licencee Section 88
a. Where the Controller is satisfied on an application for compulsory license that the manufacture, use or sale of materials not protected by the patent is prejudiced by reason of conditions imposed by the patentee upon the grant of licences under the patent, or upon the purchase, hire or use of the patented article or process, he may, subject to the provisions of Section 84, order the grant of licences under the patent to such customers of the applicant as he thinks fit as well as to the applicant. b. Where an application for compulsory license is made by a person being the holder of a licence under the patent, the Controller may, if he makes an order for the grant of a licence to the applicant, order the existing licence to be cancelled, or may, if the thinks fit, instead of making an order for the grant of a licence to the applicant, order the existing licence to be amended.
13.02.01.06 Compulsory license in case of two or more patents held by the same patentee Section 88
Where two or more patents are held by the same patentee and an applicant for a compulsory licence establishes that the reasonable requirements of the public have not been satisfied with respect to some only of the said patents, then, if the Controller is satisfied that the applicant cannot efficiently or satisfactorily work the licence granted to him under those patents without infringing the other patents held by the patentee and if those patents involve important technical advancement or considerable economic significance in relation to the other patents, he may, by
order, direct the grant of a licence in respect of the other patents also to enable the licensee to work the patent or patents in regard to which a licence is granted under section 84.
13.02.02 License for related patents Section 91
a. At any time after the grant of a patent, any person who has the right to work any other patented invention, either as patentee or as licensee thereof, exclusive or otherwise, may apply to the Controller for the grant of a licence of the first mentioned patent on the ground that he is prevented or hindered without such licence from working the other invention efficiently or to the best advantage possible. b. No order under such an application shall be made unless the Controller is satisfied – i. that the applicant is able and willing to grant, or procure the grant to the patentee and his licensees if they so desire, of a licence in respect of the other invention on reasonable terms; and ii. that the other invention has made a substantial contribution to the establishment or development of commercial or industrial activities in the territory of India. c. Controller, if satisfied, that the grounds alleged have been established by the applicant, he may make an order on such terms as he thinks fit granting a licence under the first mentioned patent and a similar order under the other patent if so requested by the proprietor of the first mentioned patent or his licensee. However, such a licence granted by the Controller shall be non-assignable except with the assignment of the Form-17
respective patents. d. The procedure as mentioned in Sections 87, 88, 89 and 90 shall apply to licences a licence granted under this provision.
13.02.03 Compulsory licence on notification by Central Government Section 92
a. If the Central Government is satisfied, in respect of any patent in force in circumstances of national emergency or in circumstances of extreme urgency or in case of public non-commercial use, that it is necessary that compulsory licenses should be granted at any time after the sealing thereof to work the invention, it may make a declaration to that effect, by notification in the Official Gazette, and thereupon the following provisions shall have effect, that is to say -i. the Controller shall, on application made at any time after the notification by any person interested, grant to the applicant a licence under the patent on such terms and conditions as he thinks fit; ii. in settling the terms and conditions of a licence granted under this section, the Controller shall endeavour to secure that the articles manufactured under the patent shall be available to the public at the lowest prices consistent with the patentees deriving a reasonable advantage from their patent rights. iii. The procedure as mentioned in Sections 83,87,88,89 and 90 shall apply in relation to the grant of such licences as they apply in relation to the grant of licences under sec. 84. iv. However, where the Controller is satisfied on consideration of the application that it is necessary Form-17
in-I. a circumstance of national emergency; or II. a circumstance of extreme urgency; or III. a case of public non-commercial use, which may arise or is required, as the case may be, including public health crises, relating to Acquired Immuno Deficiency Syndrome, Human Immune Deficiency Virus, Tuberculosis, Malaria or other epidemics, the procedure as mentioned in Section 87 shall not apply. However, the Controller shall, as soon as may be practicable, inform the patentee of the patent relating to the application for such non-application of Section 87.
13.02.04 Compulsory licence for export of patented pharmaceutical products in certain exceptional circumstances Section 92A
a. Compulsory licence shall be available for manufacture and export of patented pharmaceutical product to any country having insufficient or no manufacturing capacity in the pharmaceutical sector for the concerned product to address public health problems, provided compulsory licence has been granted by such country or such country has, by notification or otherwise, allowed importation of the patented pharmaceutical products from India. b. The Controller shall, on receipt of an application in the prescribed manner, grant a compulsory licence solely for manufacture and export of the concerned pharmaceutical product to such country under such terms and conditions as may be specified and published by him. c. The provisions of (a) and (b) shall be without prejudice Form-17
to the extent to which pharmaceutical products produced under a compulsory licence can be exported under any other provision of this Act. d. 'Pharmaceutical products' means any patented product, or product manufactured through a patented process, of the pharmaceutical sector needed to address public health problems and shall be inclusive of ingredients necessary for their manufacture and diagnostic kits required for their use.
13.02.05 Termination of compulsory license Section 94
a. Patentee or any other person deriving title or interest in the patent, may make an application for termination of compulsory licence granted under Section 84 on the ground that the circumstances that gave rise to the grant thereof no longer exist and such circumstances are unlikely to recur. b. The holder of the compulsory licence shall have the right to object to such termination. c. While considering such an application, the Controller shall take into account that the interest of the person, who had previously been granted the licence, is not unduly prejudiced.

Chapter -14

Use of patent for purposes of Government

14.01 Use of patent for the purpose of Government Section 99
An invention is said to be used for the purpose of Government if it is made, used, exercised or vended for the purposes of Central Government, a State Government or a Government undertaking.
14.02 Power of Central Government to use inventions Section 100
a. Where an invention has, before the priority date of the relevant claim of the complete specification, been duly recorded in a document, or tested or tried, by or on behalf of the government or a government undertaking, otherwise than in consequence of the communication of the invention directly or indirectly, by the patentee or by a person from whom he derives title, any use of the invention by the Central Government or any person authorized in writing by it for the purposes of government may be made free of any royalty or other remuneration to the patentee. b. If and so far as the invention has not been so recorded or tried or tested as aforesaid, any use of the invention made by the Central Government of any person authorized by it as above said, at any time after grant of the patent or in consequence of any such communication as aforesaid, shall be made upon terms as may be agreed upon either before or after the use, between the Central Government or any person authorised by Central Government and the patentee, or, as may in default of agreement be determined
c. d. e. by the High Court on a reference under Section 103. In case of any such use of any patent, the patentee shall be paid not more than adequate remuneration in the circumstances of each case, taking into account the economic value of the use of the patent. The authorisation by the Central Government in respect of an invention may be given either before or after the patent is granted and either before or after the acts in respect of which such authorisation is given or done, and may be given to any person, whether or not he is authorised directly or indirectly by the applicant or the patentee to make, use, exercise or vend the invention or import the machine, apparatus or other article or medicine or drug covered by such patent. Where an invention has been used by or with the authority of the Central Government for the purposes of government then except in case of national emergency or other circumstances of extreme urgency or for noncommercial use, the government shall notify the patentee as soon as practicable of the fact and furnish him with such information as to the extent of the use of the invention as he may, from time to time, reasonably require. Where the invention has been used for the purposes of a government undertaking, the Central Government may call for such information as may be necessary for this purpose from such undertaking. The right to make, use, exercise and vend an invention for the purposes of government shall include the right to sell on non-commercial basis, the goods which have been made in exercise of that right, and a purchaser of goods so sold, and a person claiming through him, shall have the power to deal with the goods as if the Central Government
or the person authorised by the Central Government were the patentee of the invention. f. Where in respect of a patent which has been the subject of an authorisation, there is an exclusive licensee or where such patent has been assigned to the patentee in consideration of royalties or other benefits determined by reference to the use of the invention (including payments by way of minimum royalty), the notice shall also be given to such exclusive licensee or assignor, as the case may be, and the reference to the patentee shall be deemed to include a reference to such assignor or exclusive licensee.
14.03 Rights of third parties Section 101
a. In relation to any use of a patented invention, or an invention in respect of which an application for a patent is pending, made for the purposes of government i. by the Central Government or any person authorised by the Central Government under section 100; or ii. by the patentee or applicant for the patent to the order made by the Central Government, the provisions of any licence, assignment or agreement pranted or made, between the patentee or applicant for the patent (or any person who derives title from him or from whom he derives title) and any person other than the Central Government shall be of no effect so far as those provisions— i. restrict or regulate the use for the purposes of government of the invention, or of any model, document or information relating thereto, or ii. provide for the making of payments in respect of any use of the invention or of the model, document
b. c. or information relating thereto for the purposes of government and the reproduction or publication of any model or document in connection with the said use for the purposes of government shall not been deemed to be an infringement of any copyright subsisting in the model or document. Where the patent, or the right to apply for or obtain the patent, has been assigned to the patentee in consideration of royalties or other benefits determined by reference to the use of the invention then, in relation to any use of the invention made for the purposes of government by the patentee to the order of the Central Government, subsection (3) of section 100 shall have effect as if that use were made by virtue of an authority given under that section, and any use of the invention for the purposes of government by virtue of sub-section (3) of that section shall have effect as if the reference to the patentee included a reference to the assignor of the patent, and any sum payable by virtue of that sub-section shall be divided between the patentee and the assignor in such proportion as may be agreed upon between them or as may in default of agreement be determined by the High Court on a reference under Section 103. Where by virtue of sub-section (3) of section 100, payments are required to be made by the Central Government or persons authorised under sub-section (1) of that section in respect of the use of an invention for the purposes of government, and where in respect of such patent there is an exclusive licensee authorised under his licence to use the invention for the purposes of government, such sum shall be shared by the patentee and
such licensee in such proportions, if any, as may be agreed upon between them or as may in default of agreement be determined by the High Court on a reference under section 103 to be just, having regard to any expenditure incurred by the licensee— i. in developing the said invention; or ii. in making payments to the patentees other than royalties or other benefits determined by reference to the use of the invention in consideration of the licence.
14.04 Acquisition of inventions Section 102
a. The Central Government may, if satisfied that it is necessary that an invention which is the subject of an application for a patent or a patent should be acquired from the applicant or the patentee for a public purpose, publish a notification to that effect in the Official Gazette, and thereupon the invention or patent and all rights in respect of the invention or patent shall, by force of this section, stand transferred to and be vested in the Central Government. b. Notice of the acquisition shall be given to the applicant, and, where a patent has been granted, to the patentee and other persons, if any, appearing in the register as having an interest in the patent. c. The Central Government shall pay to the applicant, or as the case may be, the patentee and other persons appearing on the register as having an interest in the patent such other compensation as may be agreed upon between the Central Government and the applicant or the patentee and other persons; or, as may, in default of agreement, be determined by the High Court on a reference under section 103 to be just having regard to the expenditure
incurred in connection with the invention and, in the case of a patent, the term thereof, the period during which and the manner in which it has already been worked (including the profits made during such period by the patentee or by his licensee whether exclusive or otherwise) and other relevant factors.
14.05 Reference of disputes to High Court Section 103
a. Any dispute as to the exercise by the Central Government or a person authorised by it of the powers conferred by section 100, or as to terms for the use of an invention for the purposes of government thereunder or as to the right of any person to receive any part of a payment made in pursuance of sub-section (3) of that section or as to the amount of compensation payable for the acquisition of an invention or a patent under section 102, may be referred to the High Court by either party to the dispute in such manner as may be prescribed by the rules of the High Court. b. In any proceedings under this section to which the Central Government is a party, the Central Government may— i. if the patentee is a party to the proceedings, petition by way of counter-claim for revocation of the patent on any ground upon which a patent may be revoked under section 64; and ii. whether a patentee is or is not a party to the proceedings, put in issue the validity of the patent without petitioning for its revocation. c. If in such proceedings as aforesaid any question arises whether an invention has been recorded, tested or tried as is mentioned in section 100, and the disclosure of any document regarding the invention, or of any evidence of the test or trial thereof, would, in the opinion of the
d. e. a. Central Government, be prejudicial to the public interest, the disclosure may be made confidentially to the advocate of the other party or to an independent expert mutually agreed upon. In determining under this section any dispute between the Central Government and any person as to terms for the use of an invention for the purposes of government, the High Court shall have regard to any benefit or compensation which that person or any person from whom he derives title, may have received, or may be entitled to receive, directly or indirectly in respect of the use of the invention in question for the purposes of government. In any proceedings under this section, the High Court may at any time order the whole proceedings or any question or issue of fact arising therein to be referred to an official referee, commissioner or an arbitrator on such terms as the High Court may direct, and references to the High Court in the foregoing provisions of this section shall be construed accordingly. Where the invention claimed in a patent was made by a person who at time it was made was in the service of the Central Government or of a State Government or was an employee of a government undertaking and the subject-matter of the invention is certified by the relevant government or the principal officer of the government undertaking to be connected with the work done in the course of the normal duties of the government servant or employee of the government undertaking, then, notwithstanding anything contained in this section, any dispute of the nature referred to in sub-section (1) relating to the invention shall be disposed of by the Central
Government conformably to the provisions of this section so far as may be applicable, but before doing so the Central Government shall give an opportunity to the patentee and such other parties as it considers have an interest in the matter to be heard.

Chapter 15 Patent Agents

15.01 Patent Agents Section 126, 127
a. A patent application can be filed and prosecuted by the Applicant himself or through a registered Indian patent agent. The Register of Patent Agents containing the names and addresses of all the registered patent agents is available at: www.ipindia.nic.in b. The Patents Act read with the Patents Rules prescribe the qualifications and the eligibility for becoming a patent agent. In order to get registered as a patent agent one has to pass an examination conducted by the Controller General of Patents annually. The notification concerning the examination is published in the official website www.ipindia.nic.in and also in at least one prominent newspaper. c. In order to apply for registration as a patent agent, one has to be a citizen of India, above the age of 21, and should have a Bachelor’s degree in Science or Engineering from a recognized Indian University or possesses such other equivalent qualifications as the Central Government may specify in this behalf. d. All matters relating registration and subsequent procedures are dealt with in the Office of The Controller General of Patents, Designs and Trademarks, Mumbai.
15.02 Disqualifications for registration as a patent agent Rule 114
A person shall not be eligible to be registered as a patent agent, if he -
a. has been adjudged by a competent court to be of unsound mind; b. is an undischarged insolvent; c. being a discharged insolvent, has not obtained from the court a certificate to the effect that his insolvency was caused by misfortune without any misconduct on his part; d. has been convicted by a competent court, whether within or outside India of an offence to undergo a term of imprisonment, unless the offence of which he has been convicted has been pardoned or unless on an application made by him, the Central Government has, by order in this behalf, removed the disability; e. being a legal practitioner has been guilty of professional misconduct; or f. being a chartered accountant, has been guilty of negligence or misconduct.
15.03 Rights of patent agents Section 127
A patent agent is entitled-a. to practise before the Controller; and b. to prepare all documents, transact all business and discharge such other functions as may be prescribed in connection with any proceedings before the Controller under this Act.
15.04 Subscription and verification of certain documents by a Patent Agent Section 128
All applications and communications to the Controller under this Act may be signed by a patent agent authorised in writing in this behalf by the person concerned.
15.05 Restrictions on Practice as Patent Agents Section 129
Only a person registered as a patent agent is authorized to practice. In the case of a partnership, the firm may be described or held out as Patent Agent, only if all of the partners of the
Firm are registered as patent agents. No company or other body corporate shall practice, describe itself or hold itself out as Patent Agents or permit itself to be so described or held out. Each person in the associate group if any constituted should be a registered Agent and duly authorized by the concerned person on behalf they act.
15.06 Power of Controller to refuse to deal with certain agents Section 131
The Controller may refuse to recognise as agent in respect of any business under this Act: a. any individual whose name has been removed from, and not restored to, the register; b. any person who has been convicted of an offence under section 123; c. any person, not being registered as a patent agent, who in the opinion of the Controller is engaged wholly or mainly in acting as agent in applying for patents in India or elsewhere in the name or for the benefit of the person by whom he is employed; d. any company or firm, if any person whom the Controller could refuse to recognise as agent in respect of any business under this Act, is acting as a director or manager of the company or is a partner in the firm. e. The Controller shall refuse to recognise as agent in respect of any business under this Act any person who neither resides nor has a place of business in India.
15.07 Power of Controller to remove the name of a Patent Agent
a. The name of any person from the Register can be removed if the Controller is satisfied that: – i. his/her name has been entered in the Register by error on account of misrepresentation or suppression of material fact; or ii. he/she has been convicted of any offence and
sentenced to a term of imprisonment or has been guilty of misconduct in his professional capacity which in the opinion of the Controller renders him unfit to be kept in the register. b. The Controller shall take such decision after giving that person a reasonable opportunity of being heard and after any further inquiry, as he thinks fit to make. c. Besides, the name of the Patent agent can be removed also when: i. request has been received from the Patent Aagent to that effect; or ii. he/she is dead; or iii. if he/she has defaulted in the payment of fees specified in the First Schedule by more than three months after they are due. d. The decision of the removal of the name of any person from the Register of Patent Agents shall be published and will be communicated to the person concerned.
15.08 Restoration of names of Patent Agents Rule 117
a. Restoration of names of persons removed from the register of Patent Agents can be made by the Controller, on Application made in form 23 within two months from the date of such removal. b. The restoration of name to the register shall be published and communicated to the person concerned. c. If the name of a person is entered in the register of Patents Agents, his name shall be continued therein for a period of one year from the date on which his last annual fee became due.

Chapter 16 Offences and Penalties

16.01 Contravention of S.35 or 39 Section 118
If any person fails to comply with any direction given under section 35 or makes or causes to be made an application for the grant of a patent in contravention of section 39, he shall be punishable with imprisonment for a term which may extend to two years, or with fine, or with both.
16.02 Falsification of entries in register, etc. Section 119
If any person makes a false entry in any register kept under the Patents Act or provides any writing or evidence as a result of which the entry in the register results into a false entry, then he is punishable with imprisonment for a term that may extend to 2 years or with fine or with both.
16.03 Unauthorized Claim of Patent Rights a. If any person falsely represents that any article sold by him is patented in India or is the subject of an Application for a Patent in India, he shall be punishable with fine that may extend to rupees one lakh . b. The use of words 'Patent', Patented', 'Patent applied for', 'Patent pending', without mentioning the name of the country means that they are patented in India or Patent applied for in India and attract the provisions of this section Section 120
16.04 Wrongful use of words, "Patent Office" If any person uses on his place of business or any document Section 121
issued by him which would reasonably lead to the belief that either his place of business is the Patent Office or is officially connected with the Patent Office, he shall be punishable with imprisonment for a term that may extend to 6 months, or with fine, or with both.
16.05 Refusal or failure to supply information If any person refuses or fails to furnish information as required under Sections 100(5) and 146 he shall be punishable with fine which may go up to rupees one lakh. Section 100(5) provides that any person including Government undertaking using a patented invention for the purpose of Government has to furnish any information on the use of invention as required by the Central Government and Section 146 provides that the patentee has to furnish a statement regarding the working of the patented invention in a commercial scale in India in Form 27. This has to be done annually within 3 months of the end of each year. If he furnishes false information knowingly he shall be punishable with imprisonment that may extend to 6 months or with fine or with both. Section 122
16.06 Practice by non-registered persons Any person practicing as a Patent Agent without registering himself as such is liable to be punished with a fine of rupees one lakh for the first offence and rupees five lakh for subsequent offence. Section 123
16.07 Offences by Companies When an offence is committed by a company, the company as well as every person such as director, manager, secretary, in charge of, and responsible to the company for the conduct of its business at the time of the commission of the offence , shall be deemed to be guilty and shall be liable to be proceeded against and punished accordingly. The punishment will not be applicable if such person that may include director Section 124

including partner, manager, secretary, proves that the offence was committed without his knowledge or he has taken due care to prevent the offence.

Chapter-17 General powers of Controller

17.01 Powers of a Civil Court Section 77
Subject to any rules made in this behalf, the Controller in any proceedings before him under this Act have the powers of a civil court while trying a suit under the Code of Civil Procedure, 1908 (5 of 1908) in respect of the following matters, namely,— a. summoning and enforcing the attendance of any person and examining him on oath; b. requiring the discovery and production of any document; c. receiving evidence on affidavits; d. issuing commissions for the examination of witnesses of documents; e. awarding costs. f. reviewing his own decision on application made within the prescribed time and in the prescribed manner; g. setting aside an order passed ex parte on application made within the prescribed time and in the prescribed manner; h. any other matter which may be prescribed. Fourth Schedule Rule 136 CPC Rule 47
17.02 Awarding costs Section 77(1)(e) Rule 63, 136
a. Any order for costs awarded by the Controller in exercise of the powers conferred upon him is executable as a decree of a civil court. In all proceedings before the Controller, costs may be awarded by the Controller, as he considers
reasonable, having regard to all the circumstances of the case. b. However, the amount of costs awarded in respect of any matter set forth in the Fourth Schedule shall not exceed the amount specified therein. c. Notwithstanding anything contained in (a) above, the Controller may, in his discretion award a compensatory cost in any proceeding before him which in his opinion is false or vexatious.
17.03 Review
a. Any person considering himself aggrieved by any order or decree of the Controller from which an appeal is allowed but no appeal has been preferred, or from which no appeal is allowed, and who, from the discovery of new and important matter or evidence which, after the exercise of due diligence was not within his knowledge or could not be produced by him at the time when the order or decree was passed or order made, or on account of some mistake or error apparent on the face of the record or for any other sufficient reason, desires to obtain a review of the decree passed or order made against him, may apply for a review of the order or decree to the Controller. b. An application to the Controller for the review of his decision under clause (f) of sub-section (1) of section 77 shall be made within one month from the date of communication of such decision to the applicant or within such further period not exceeding one month thereafter as the Controller may on a request allow. c. An application for review shall be accompanied by a statement setting forth the grounds on which the review is sought. Section 114, Order XLVII of CPC, 1908 Section 77(1)(f) Form-24 Form-4
d. Where the decision in question concerns any other person in addition to the applicant, the Controller shall forthwith transmit a copy of each of the application and the statement to the other person concerned.
e. An application to the Controller for setting aside an Section 77(1)(f)
order passed by him exparte under clause (g) of sub Form-24
section (1) of section 77 shall be made within one month from the date of communication of such order to the applicant or within such further period not exceeding one month as the Controller may on a request allow and shall be accompanied by a statement setting forth the grounds on which the application is based. Where the order concerns any other person in addition to the applicant, the Controller shall, forthwith transmit a copy each of the application and the statement to the other person concerned. Form 4
17.04 Petition for obviating an irregularity Rule 137
Any document for the amendment of which no special provision is made in the Act may be amended and any irregularity in procedure which in the opinion of the Controller may be obviated without detriment to the interests of any person, may be corrected if the Controller thinks fit and upon such terms as he may direct. While considering a petition under Rule 137, only such an irregularity is allowed to be obviated which is without detriment to the interests of any person. Further, only such amendments for which there is no special provision in the Act and which may be made without detriment to the interests of any person are allowable. Generally, a failure to act within prescribed time may not be considered as an irregularity, which can be obviated under this Rule.
17.05 Mention of inventor in Patent Section 28
a. b. c. d. e. f. g. If the controller is satisfied, upon a request or claim made in accordance with the provisions of this section— i) that the person in respect of or by whom the request or claim is made is the' inventor of an invention in respect of which application for a patent has been made, or of a substantial part of that invention; and ii) that the application for the patent is a direct consequence of his being the inventor, the Controller shall, subject to the provisions of this section, cause him to be mentioned as inventor in any patent granted in pursuance of the application in the complete specification and in the register of patents: Such a request or claim shall be accompanied by a statement setting out the circumstances under which the claim is made. However, the mention of any person as inventor under this section shall not confer or derogate from any rights under the patent. A request that any person shall be mentioned as aforesaid may be made in the prescribed manner by the applicant for the patent or (where the person alleged to be the inventor is not the applicant or one of the applicants) by the applicant and that person. If any person [other than a person in respect of whom a request in relation to the application in question has been made under sub-section (2),] desires to be mentioned as aforesaid, he may make a claim in the prescribed manner in that behalf. A request or claim under the foregoing provisions of this section shall be made before the grant of patent. Where such a claim is made, the Controller shall give notice of the claim to every applicant for the patent (not Form 8
being the claimant) and to any other person whom the Controller may consider to be interested and before deciding upon any such request or claim, the Controller shall, if required, hear the person in respect of or by whom the request or claim is made, and also any person to whom notice of the claim has been given as aforesaid. h. Where any person has been mentioned as inventor in pursuance of such an application, any other person who alleges that he ought not to have been so mentioned may at any time apply to the Controller for a certificate to that effect, and the Controller may, after hearing, if required, any person whom he may consider to be interested, issue such a certificate, and if he does so, he shall rectify the specification and the register accordingly. i. The procedure specified in rules 55A and 57 to 63 relating to the filing of notice of opposition, written statement, reply statement, leaving evidence, hearing and cost shall, so far as may be, apply to the hearing of such a claim or application as they apply to the opposition proceedings subject to the modification that reference to patentee shall be construed as the person making the claim, or an application, as the case may be. j. Any mention of the inventor under sub-section (1) of section 28 shall be made in the relevant documents in the following form namely:- "The inventor of this invention/substantial part of this invention within the meaning of section 28 of the Patents Act, 1970, is…of…”.
17.06 Directions not otherwise prescribed Rule 128
a. Where for the proper prosecution or completion of any
proceedings under the Act or these rules, the Controller is of the opinion that it is necessary for a party to such proceedings to perform an act, file a document or produce evidence, for which provision has not been made in the Act or these rules, he may, by notice in writing, require such party to perform the act, file the document or produce the evidence specified in such notice. b. Where an applicant or a party to a proceeding desires to be heard or not heard, the Controller may, at any time, require him to submit his statement in writing giving such information as the Controller may deem necessary within the time specified by him.
17.07 Exercise of discretionary power by the Controller
a. Before acting adverse to any party, the Controller shall give an opportunity of being heard to the party. The discretionary powers shall be exercised with due care and caution and not in an arbitrary manner. Such reasons shall be taken judiciously and the reasons shall be recorded in the file. However, this will not apply to actions resulting from “deemed” provisions in the Act and Rules. b. A party desiring a hearing shall make the request for such hearing to the Controller at least ten days in advance of the expiry of the time-limit specified in respect of the proceeding. c. Before exercising any discretionary power under the Act or these rules which is likely to affect an applicant for a patent or a party to a proceeding adversely, the Controller shall give such applicant or party, a hearing, after giving him or them, ten days' notice of such hearing ordinarily. Section 80 Rule 129
17.08 Power of Controller to correct clerical errors, etc. Section 78
a. Without prejudice to the provisions contained in sections 57 and 59 as regards amendment of applications for patents or complete specifications or other documents relating thereto and subject to the provisions of section 44, the Controller may, in accordance with the provisions of this section, correct any clerical error in any patent or in any specification or other document filed in pursuance of such application or in any application for a patent or any clerical error in any matter which is entered in the register. b. A correction may be made in pursuance of this section either upon a request in writing made by any person interested and accompanied by the prescribed fee, or without such a request. c. Where the Controller proposes to make any such correction as aforesaid otherwise than in pursuance of a request, he shall give notice of the proposal to the patentee or the applicant for the patent, as the case may be, and to any other person who appears to him to be concerned, and shall give them an opportunity to be heard before making the correction. d. Where a request is made for the correction of any error in a patent or application for a patent or any document filed in pursuance of such an application, and it appears to the Controller that the correction would materially alter the meaning or scope of the document to which the request relates and ought not to be made without notice to persons affected thereby, he shall require notice of the nature of the proposed correction to be published in the official journal. Rule 22
e. Such a request for the correction of a clerical error in any document shall be accompanied by a copy of the document highlighting the corrections clearly along with the prescribed fees. f. Within the prescribed time after any such publication as aforesaid any person interested may give notice to the Controller of opposition to the request, and, where such notice of opposition is given, the Controller shall give notice thereof to the person by whom the request was made, and shall give to him and to the opponent an opportunity to be heard before he decides the case. g. The procedure specified in rules 58 to 63 relating to the filing of reply statement, leaving evidence, hearing and costs shall, so far as may be, will be applicable to the above proceedings.
17.09 Ex-parte decision Before proceeding ex-parte against any party, the Controller shall issue a notice to the concerned party clearly stating therein that if the party fails to attend the hearing so fixed, he shall be proceeded ex-parte. Such notice shall be sent by Registered Post with Acknowledgment due.

Chapter 18 General Services

18.01 General Services
a. Patent Office provides certain statutory and non- statutory services for the dissemination of information related to patent processing. b. It may be noted that references to some of these services have already been made in the relevant Chapters. c. These services are enumerated in the following paragraphs with the standards set by the Office.
18.02 Official Journal Section 145
a. Every Friday the Controller publishes the Official Patent Journal electronically and is made available on the official website. b. The journal contains the following information: i. 11A publication, including early publication. ii. Withdrawal of Patent Applications. iii. Cessation of Patents. iv. Restoration of lapsed Patents. v. Post-Grant amendments. vi. Amendment of Patent granted to deceased applicant. vii. Assignment after grant. viii. Post-Grant Oppositions. ix. Working of Patents. x. Revocation of Patents. xi. Compulsory Licences. xii. Rectification of register by Aappellate Board. xiii. Details of Government use. xiv. General matters. xv. Surrender of patents.
c. Publications under the Designs Act appears as the last part in the journal.
18.03 Information relating to patent applications and patents
a. At the request of a person (on plain paper) the Controller provides following information regarding a Patent or an Application for a Patent. Separate requests shall be made in respect of each item of information: i. as to when a complete Specification following a Provisional Specification has been filed or an Application for Patent has been deemed to have been abandoned; ii. as to when the information under Section 8 has been filed; iii. as to when Publication of Application has been made under section 11 A; iv. as to when an Application has been withdrawn under Section 11B; v. as to when a request for examination has been made under Section 11B; vi. as to when the examination report has been issued under Section 12; vii. as to when an Application for Patent has been refused; viii. as to when a Patent has been granted; ix. as to when a renewal fee has been paid; x. as to when the term of a Patent has expired or shall expire; xi. as to when an entry has been made in the Register or Application has been made for the making of such entry; or xii. as to when any Application is made or action taken involving an entry in the Rregister, Section 153. Rule 134.
publication in the Official Journal or otherwise, if the nature of the Application or action is specified in the request. b. The report of examiner to the Controller under Section 12 is not open to public unless directed a Court of Law. c. Except (b) above, most of the information relating to patents is available through the official website. But, the website information will not be sufficient for legal purposes, for which a person may take recourse to (a) above.
18.04 Inspection and supply of copies of documents Rule 27, 74A
a. After the publication of application, the application Section 72,
together with the complete specification, provisional 147.
specification, drawing, if any and abstract may be Rule 27,
inspected at the appropriate office. b. After the grant of a patent the application together with the complete specification, provisional specification, drawing, if any, and abstract and related thereto may be inspected at the appropriate office. c. Request for inspection may be made in plain paper along with the prescribed fee. d. A person may obtain copies of any document open to public on payment of the prescribed fee. e. Certified copy (as may be required for legal purposes) of any document open to public may be obtained on payment of prescribed fee. f. Register of Patents may be inspected within the working hours of the Patent Office on making a written application on plain paper along with the prescribed fee. g. Certified copies of any entry in the register is available at the payment of prescribed fee. 74A.
h. If a Certificate of Patent is lost or destroyed, an Section 154
application for a duplicate Certificate of Patent may be made on a plain paper. Rule 132
18.05 Annual report of the Patent Office Section 155
The Office publishes an annual report comprising statistical information regarding the activities of this Office. Such report is placed before both the Houses of Parliament, whereupon the report is made available on the official website.
18.06 Information available at the Website
The official website provides the following information: a. Indian Patent Information Retrieval System (IPIRS) provides information on: a. Granted Patents; b. Published Patent Applications; c. Controller’s decisions; and d. Patent Application status. b. The objective of the IPIRS is to facilitate search on Indian Patents / Patent Applications for the purpose of novelty and inventive step and to enable applicant or any other person to procure information about the status of a Patent Application. c. List of registered Patent Agents. d. Examination schedule for Patent Agents. e. Office orders / circulars by the Controller General are made available on the website.

Chapter 19 Scientific Advisors

19.01 Scientific Advisors Section 115
a. In any suit for infringement or in any proceeding before a court under this Act, the court may at any time, and whether or not an application has been made by any party for that purpose appoint an independent scientific adviser to assist the court or to inquire and report upon any such question of fact or of opinion (not involving a question of interpretation of law) as it may formulate for the purpose. b. The remuneration of the scientific adviser shall be fixed by the court and shall include the costs of making a report and a proper daily fee for any day on which the scientific adviser may be required to attend before the court, and such remuneration shall be defrayed out of moneys provided by Parliament by law for the purpose.
19.02 Roll of Scientific Advisors Rule 103
The Controller maintains a roll of scientific advisers for the purpose of section 115. The roll is updated annually. The roll contains the names, addresses, specimen signatures and photographs of scientific advisers, their designations, information regarding their educational qualifications, the disciplines of their specialisation and their technical, practical and research experience. The roll is available on the official website www.ipindia.nic.in
19.03 Qualifications and procedure for enrollment Rule 103, 104
a. Any person who— i. holds a degree in science, engineering, technology or equivalent; ii. has at least fifteen years' practical or research
experience; and iii. he holds or has held a responsible post in a scientific or technical department of the Central or State Government or in any organization, is qualified to have his name entered in the roll of scientific advisers. However, where the Controller is of the opinion that it is necessary or expedient so to do, he may, by order, for reasons to be recorded in writing, relax any of the qualifications specified above, with respect to any person, if such person is otherwise well qualified. b. Any interested person may apply at any time to the Controller for inclusion of his name in the roll of scientific advisers furnishing his bio-data. c. The roll is updated and published annually.
19.04 Removal from the roll Rule 104
a. The Controller may remove the name of any person from the roll of scientific advisers, if— i. such person makes a request for such removal; or ii. the Controller is satisfied that his name has been entered in the roll by error or account of misrepresentation or suppression of any material fact; or iii. such person has been convicted of an offence and sentenced to a term of imprisonment or has been guilty of misconduct in his professional capacity and the Controller is of the opinion that his name should be removed from the roll: b. Before removing the name of any person from the roll of scientific advisers under this rule, such person shall be given a reasonable opportunity of being heard.

Chapter 20 Miscellaneous provisions

20.01 Affidavits
a. In any proceeding under this Act before the Controller, evidence shall be given by affidavit in the absence of directions by the Controller to the contrary. b. In any case in which the Controller thinks it right so to do, he may take oral evidence in lieu of, or in addition to, evidence by affidavit, or may allow any party to be cross-examined on the contents of his affidavit. c. The affidavits required by the Act or these rules to be filed at the patent office or furnished to the Controller shall be duly sworn to in the manner as prescribed. d. Affidavits shall be confined to such facts as the deponent is able, of his own knowledge, to prove except in interlocutory matters, where statements of belief of the deponent may be admitted, provided that the grounds thereof are given. e. Affidavits shall be sworn to as follows: i) in India - before any court or person having by law authority to receive evidence, or before any officer empowered by such court as aforesaid to administer oaths or to take affidavits; ii) in any country or place outside India - before a diplomatic or consular officer, within the meaning of the Diplomatic and Consular Officers (Oaths and Fees) Act, 1948 (41 of 1948) in such country or place or before a notary of the country or place, recognised Section 79, Rule 126. Diplomatic and Consular Officers (Oaths and Fees) Act, 1948. Notifications u/s 14 of the Notaries Act, 1952.
by the Central Government under section 14 of the Notaries Act, 1952 (53 of 1952), or before a Judge or Magistrate of the country or place. f. Alterations and interlineations shall, before an affidavit is sworn to or affirmed be authenticated by the initials of the person before whom the affidavit is sworn to.
20.02 Exhibits Rule 127
Where there are exhibits to be filed in an opposition or any other proceedings, a copy or impression of each exhibit shall be supplied to the other party at his request and expense; if copies or impressions of the exhibits cannot conveniently be furnished, the originals shall be left with the Controller for inspection by the person interested by prior appointment. The exhibits in original if not already left with Controller shall be produced at the hearing.
20.03 Officers and employees of Patent Office – duties Section 76
An officer or employee in the patent office shall not, except when required or authorised by this Act or under a direction in writing of the Central Government or Appellate Board or the Controller or by order of a court— a. furnish information on a matter which is being, or has been, dealt with under this Act; or b. prepare to assist in the preparation of a document required or permitted by or under this Act to be lodged in the patent office; or c. conduct a search in the records of the patent office.
20.04 Hearing to be in public Rule 139
Where the hearing before the Controller of any dispute between two or more parties relating to an application for a patent or to any matter in connection with a patent takes place after the date of the publication of the complete
specification, the hearing of the dispute shall be in public unless the Controller, after consultation with the parties to the dispute who appear in person or are represented at the hearing, otherwise directs.
20.05 Agency – Power of Attorney Rule 135
a. The authorisation of an agent for the purposes of the Act and these rules shall be in Form 26 or in the form of a power of attorney. b. Where any such authorisation has been made, service upon the agent of any document relating to any proceeding or matter under the Act or these rules shall be deemed to be service upon the person so authorising him and all communications directed to be made to a person in respect of any proceeding or matter may be addressed to such agent, and all appearances before the Controller relating thereto may be made by or through such agent. c. If it is considered necessary, require the personal signature or presence of an applicant, opponent or party to such proceeding or matter. d. As a matter of practice, an application may be accepted without a power of attorney, for the purpose of saving priority. However, any subsequent papers can be filed only after submitting the required power of authority. No office action will be done without having a Form-26 / authorisation of agent on record. In cases where a General Power of Attorney from an applicant has already been filed in the Office, a copy attested by the agent shall be filed for subsequent filing. Form-26

Chapter 21 Time Limits

21.01 Time limits
Time limits for various actions while prosecuting the patent application and also for post grant procedures have been either specifically provided in the Patents Act or prescribed through the Patents Rules. These time limits are required to be followed strictly by every person concerned. Failure to adhere to the legally imposed time limits may turn out to be detrimental to the interests of the applicants, patentees or any other person interested.
21.02 Petition for extension of time Rule 138
Time limits prescribed in the Rules for doing of any Act or taking of any proceedings thereunder may be extended by the Controller for a period of one month, if he thinks it fit to do so and upon such terms as he may direct. He may do so based on a petition under Rule 138. However, the time limits prescribed under Rules 24B, 55(4) and 80(1A) are not extendable under this provision. Any request for extension made under the Rules shall be made before the expiry of the prescribed period. As Rule 138 specifically stipulates that any time limit prescribed by the Patents Rules can be extended by one month only, any further extension shall not be allowable by invoking any other provision, such as Rule 137, except wherever specifically allowed by the Act or Rules viz. Rules 13(6), 80(1A), 130.

21.03: Time limits prescribed by the Patents Act, 1970 and Patents Rules, 2003

Description Time Provision
1. Proof of right to make an application Six months from the date of filling of application Section 7(2) (Rule10)
2. Statement and undertaking regarding foreign applications Six months from the date of filing of application Section 8(1) Rule 12(1A)
3. Subsequent information corresponding to foreign filing Six months from the date of filing of application outside India Section 8(1)(b) Rule 12(2)
4. Information relating to objections in respect of novelty, patentability etc. in foreign filing Six months from the date of communication by Controller Section 8(2) Rule 12(3)
5. Filing a complete specification after filing provisional specification Twelve months from the date of filling of the Provisional Specification Section 9(1)
6. Declaration of Inventorship (Form 5) With the complete specification or within one month from the date of filing of the complete specification Rule 13(6)
7. Reference to deposit of biological material Three months form the date of filing of application Section10(4)(ii)A Rule 13(8)
8. Convention application Twelve months from the date of filing of basic application Section 135(1)
9. Convention application (in case of multiple priorities) Twelve months from the date of filing of first filed basic application Section 135(1)
10. Convention application (cognate) Twelve months from the date of earliest filed specification Section 135(2)
11. PCT national phase application Thirty one months from the priority date Rule 20(4)(i)
12. Priority document (for convention application) Three months from the date of communication from the Controller Section 138(1) Rule 121
13. Publication of application Ordinarily within one month from the expiry of eighteen months from the date of filing or priority or one month from the date of request for early publication, whichever is earlier Rule 24, 24A
Description Time Provision
14. Withdrawal of application to prevent publication Fifteen months from date of filing or priority, whichever is earlier Sec 11A(3)(c)
15. Request for examination Forty eight months from the date of filing or priority, whichever is earlier Section 11B, Rule 24B
16. Request for examination, where secrecy direction imposed Forty eight months from the date of filing or priority or sixth months from the date of revocation of secrecy direction, whichever expires later Rule 24B(1)(iii)
17. Request for examination (Divisional Application) Forty eight months from date of filing or priority of first mentioned application, or within six months from date of filing of further application, whichever expires later Rule 24B(1)(iv)
18. Request for withdrawal Any time before the grant of patent Sec 11B(4), Rule 26
19. Request for examination received, Controller refers the Application to the examiner Ordinarily within one month from the date of publication or request for examination, whichever is later Rule 24B(2)(i)
20. Time within which Examiner makes report to Controller Ordinarily within one month but not exceeding three months from the date of such reference Rule 24B(2)(ii)
21. Controller disposes off the report of Examiner Ordinarily within one month from the date of receipt of report Rule 24B(2)(iii)
22. First Examination Report (FER) sent by Controller to applicant Ordinarily within six months from request for examination or publication, whichever is later Rule 24B(3)
23. Time for complying with all requirements imposed by the Act Twelve months the date of FER Rule 24B(4)
24. Time, after publication, before expiry of which no patent is granted Six months from the date of publication Rule 55(1)(A)
Description Time Provision
25. Pre-grant opposition Any time before the grant of patent Section 25(1)
26. Reply statement and evidence (pregrant opposition) Three months from the date of notice of Controller Rule 55(4)
27. Decision by Controller upon pre-grant opposition Ordinarily within one month from completion of proceeding Rule 55(6)
28. Notice of Opposition (post-grant opposition) One year from the date of publication of grant of patent Sec 25(2)
29. Reply statement by patentee Two months from receipt of opponent’s written statement Rule 58(1)
30. Reply evidence by opponent One month from date of delivery of patentee’s reply statement Rule 59
31. Opposition Board submits report Three months from the date on which documents were forwarded to Board Rule 56(4)
32. Periodical review of secrecy directions Every six months Sec 36(1)
33. Controller disposes permission for filing abroad Ordinarily within twenty one days from such request Section 39 Rule 71
34. Time after which no permission required for filing abroad Six weeks after filing the application in India, where no direction for secrecy in present Section 39 (1)
35. First renewal fee In respect of third year, before the expiry of second year Rule 80(1)
36. Payment of first renewal fee, where patent granted after the expiry of two years from date of filing Three months from date of recording in register of patents Section 142(4)
37. Extension in time for payment of renewal fee, where patent granted after expiry of two years from date of filing Extendable at the most by six months Section 142(4)
38. Time for payment of the renewal fee Before the expiry of the nth year from date of patent in respect of the (n+1) th year Rule 80(1)
Description Time Provision
39. Extension in time for payment of renewal fee Maximum six months Rule 80(1A)
40. Application for restoration of patent Eighteen months from the date on which Patent ceased to have effect Section 60
41. Request for hearing by applicant for restoration, where prima facie case not made out One month from date of intimation by Controller Rule 84(2)
42. Notice of Opposition against restoration Two months from the date of publication of application for restoration Rule 85(1)
43. Payment of the unpaid renewal few and additional fee when restoration allowed One month from date of order Rule 86(1)
44. Notice of Opposition against an offer to surrender a patent Three months from the date of publication of offer Rule 87(2)
45. Notice of Opposition against application for post-grant amendment Three months from the date of publication of such an application Rule 81(3)(b)
46. Furnishing information relating to working of patent in respect of calendar year Three months from the end of each year Section 146(2), Rule 131 (2)
47. Furnishing information relating to working of patent, upon notice of Controller Two months from the date of notice Section 146(1)